casimedicos-exp / data /fr /fr.dev_casimedicos.jsonl
ragerri's picture
Upload 12 files
ef95312 verified
raw
history blame
No virus
158 kB
{"id": 274, "year": 2016, "question_id_specific": 73, "full_question": "Un homme de 52 ans, sans maladie concomitante, se présente au service des urgences avec un méléna depuis 24 heures sans répercussion hémodynamique. Il nie prendre des anti-inflammatoires non stéroïdiens. L'hématocrite était de 33 % et les autres analyses sanguines étaient normales. L'endoscopie digestive haute réalisée en urgence 6 heures après l'admission montre un estomac normal, sans sang ni débris hématiques et un ulcère excavé de 8 mm de diamètre sur la face antérieure du bulbe duodénal avec un \"vaisseau visible\" à sa base et sans saignement actif. Laquelle des affirmations suivantes est vraie ?", "full_answer": "Il s'agit d'un ulcère de grade IIa (classification de Forrest), avec un risque élevé de récidive. C'est pourquoi un traitement endoscopique et une hospitalisation avec traitement intraveineux aux IPP (généralement l'oméprazole) pendant au moins 72 heures sont clairement indiqués. La chirurgie est indiquée en cas d'hémorragie massive ou d'hémorragie réfractaire au traitement médical.", "type": "SYSTÈME DIGESTIF", "options": {"1": "Un traitement endoscopique est indiqué lors de l'endoscopie initiale, suivi d'un traitement intraveineux avec de fortes doses d'inhibiteur de la pompe à protons. Il a été démontré que cette stratégie réduisait le risque de récidive hémorragique et de mortalité.", "2": "Lors de l'endoscopie initiale, en l'absence d'hémorragie active, le traitement endoscopique n'est pas indiqué. Par la suite, pour réduire le risque de récidive hémorragique, un traitement intraveineux avec de fortes doses d'inhibiteur de la pompe à protons doit être mis en place.", "3": "Lors de l'endoscopie initiale, un traitement endoscopique est indiqué. Il n'a pas été démontré qu'un traitement ultérieur par de fortes doses d'inhibiteurs de la pompe à protons apportait un bénéfice supplémentaire.", "4": "Comme il s'agit d'un ulcère compliqué (hémorragie), la meilleure option thérapeutique, une fois l'épisode hémorragique résolu, est une vagotomie et une pyloroplastie.", "5": null}, "correct_option": 1, "explanations": {"1": {"exist": true, "char_ranges": [[0, 280]], "word_ranges": [[0, 39]], "text": "Il s'agit d'un ulcère de grade IIa (classification de Forrest), avec un risque élevé de récidive. C'est pourquoi un traitement endoscopique et une hospitalisation avec traitement intraveineux aux IPP (généralement l'oméprazole) pendant au moins 72 heures sont clairement indiqués."}, "2": {"exist": false, "char_ranges": [], "word_ranges": [], "text": ""}, "3": {"exist": false, "char_ranges": [], "word_ranges": [], "text": ""}, "4": {"exist": true, "char_ranges": [[281, 385]], "word_ranges": [[39, 53]], "text": "La chirurgie est indiquée en cas d'hémorragie massive ou d'hémorragie réfractaire au traitement médical."}, "5": {"exist": false, "char_ranges": [], "word_ranges": [], "text": ""}}}
{"id": 233, "year": 2014, "question_id_specific": 104, "full_question": "Chez une femme de 29 ans, les paramètres suivants ont été trouvés lors d'une analyse sanguine de routine : Hb 11,5 g/dL, MCV 70 fl, MCH 28 pg, Ferritine 10 ng/mL, leucocytes 5 200/mm3, plaquettes 335 000/mm3. L'examen physique est normal. L'examen le plus indiqué dans cette situation est le suivant :", "full_answer": "La cause la plus fréquente d'anémie ferriprive microcytaire chez les jeunes femmes est la perte gynécologique chronique.", "type": "HÉMATOLOGIE", "options": {"1": "Examen gynécologique.", "2": "Test de sang occulte dans les selles.", "3": "Étude radiologique du système digestif.", "4": "Électrophorèse de l'hémoglobine.", "5": "Test de Cooms."}, "correct_option": 1, "explanations": {"1": {"exist": true, "char_ranges": [[0, 120]], "word_ranges": [[0, 17]], "text": "La cause la plus fréquente d'anémie ferriprive microcytaire chez les jeunes femmes est la perte gynécologique chronique."}, "2": {"exist": false, "char_ranges": [], "word_ranges": [], "text": ""}, "3": {"exist": false, "char_ranges": [], "word_ranges": [], "text": ""}, "4": {"exist": false, "char_ranges": [], "word_ranges": [], "text": ""}, "5": {"exist": false, "char_ranges": [], "word_ranges": [], "text": ""}}}
{"id": 315, "year": 2016, "question_id_specific": 136, "full_question": "Un homme de 41 ans s'est présenté au service des urgences après trois jours d'œdème et de douleur au genou droit, avec impuissance fonctionnelle et fièvre. Deux semaines auparavant, il avait eu une diarrhée auto-infligée. L'examen ayant révélé un épanchement articulaire, une arthrocentèse a été pratiquée et 50 cc de liquide trouble ont été obtenus, avec une viscosité réduite et les paramètres analytiques suivants : leucocytes 40.000/microL (85 % de neutrophiles), glucose 40 mg/dL, absence de cristaux, coloration de Gram : aucun micro-organisme n'a été observé. Laquelle des affirmations suivantes concernant ce patient est FAUSSE ?", "full_answer": "Un Gram négatif ou une culture négative n'excluent jamais l'option d'une étiologie infectieuse, encore moins dans le cas d'une éventuelle arthrite septique. Tous les autres cas sont vrais.", "type": "RHEUMATOLOGIE", "options": {"1": "Un traitement à base de cloxacilline et de ceftriaxone doit être mis en place en attendant les résultats de la culture du liquide.", "2": "Une aitrocentèse quotidienne est conseillée pour soulager les symptômes et prévenir la destruction des articulations.", "3": "Si la culture est négative, il s'agit probablement d'une arthrite réactive.", "4": "Une coloration de Gram négative exclut la possibilité d'une arthrite septique.", "5": null}, "correct_option": 4, "explanations": {"1": {"exist": false, "char_ranges": [], "word_ranges": [], "text": ""}, "2": {"exist": false, "char_ranges": [], "word_ranges": [], "text": ""}, "3": {"exist": false, "char_ranges": [], "word_ranges": [], "text": ""}, "4": {"exist": true, "char_ranges": [[0, 156]], "word_ranges": [[0, 22]], "text": "Un Gram négatif ou une culture négative n'excluent jamais l'option d'une étiologie infectieuse, encore moins dans le cas d'une éventuelle arthrite septique."}, "5": {"exist": false, "char_ranges": [], "word_ranges": [], "text": ""}}}
{"id": 235, "year": 2014, "question_id_specific": 107, "full_question": "Un homme de 71 ans présente une pancytopénie sévère sans présence de cellules immatures et une étude de la moelle osseuse évoquant une anémie aplastique sévère. Quelle serait l'approche thérapeutique fondamentale ?", "full_answer": "Anémie aplasique sévère et n'est pas candidat à une greffe allogénique en raison de son âge. Les antisuppresseurs semblent être les plus appropriés.", "type": "HÉMATOLOGIE", "options": {"1": "Traitement par méthylprednisolone à la dose de 1 g/kg/jour pendant 5 jours.", "2": "Étude des frères et sœurs et, s'ils sont compatibles HLA, greffe allogénique de progéniteurs hématopoïétiques.", "3": "Traitement immunosuppresseur par la ciclosporine et l'immunoglobuline antithymocytaire.", "4": "Soutien à l'hémothérapie.", "5": "Chimiothérapie et, en cas de réponse, autogreffe de cellules souches hématopoïétiques."}, "correct_option": 3, "explanations": {"1": {"exist": false, "char_ranges": [], "word_ranges": [], "text": ""}, "2": {"exist": false, "char_ranges": [], "word_ranges": [], "text": ""}, "3": {"exist": true, "char_ranges": [[0, 148]], "word_ranges": [[0, 23]], "text": "Anémie aplasique sévère et n'est pas candidat à une greffe allogénique en raison de son âge. Les antisuppresseurs semblent être les plus appropriés."}, "4": {"exist": false, "char_ranges": [], "word_ranges": [], "text": ""}, "5": {"exist": false, "char_ranges": [], "word_ranges": [], "text": ""}}}
{"id": 429, "year": 2018, "question_id_specific": 114, "full_question": "Un homme de 34 ans consulte aux urgences pour dysurie et brûlure au niveau du méat urétral, avec présence d'un écoulement épais et blanchâtre qui sort du méat depuis trois jours. Laquelle des réponses suivantes est fausse ?", "full_answer": "Un test microbiologique de confirmation de la guérison n'est pas nécessaire pour les personnes ayant reçu un diagnostic de gonorrhée urogénitale ou rectale non compliquée et qui sont traitées avec l'un des schémas recommandés ou alternatifs ; toutefois, toute personne atteinte de gonorrhée pharyngée et traitée avec un schéma alternatif doit revenir 14 jours après le traitement pour un test de guérison utilisant une culture ou un test d'amplification de l'acide nucléique (TAAN). Si le test d'amplification de l'acide nucléique est positif, une culture de confirmation doit être tentée avant le retraitement. Un antibiogramme doit être réalisé sur toutes les cultures positives. Les symptômes qui persistent après le traitement doivent être évalués par culture pour N. gonorrhoeae (avec ou sans HAT concomitante), et tout isolat gonococcique doit faire l'objet d'un test de sensibilité aux antimicrobiens (modifié d'après Sexually Transmitted Diseases Treatment Guidelines, 2015).", "type": "MALADIES INFECTIEUSES ET MICROBIOLOGIE", "options": {"1": "Un échantillon de l'exsudat urétral doit être prélevé pour établir un diagnostic étiologique.", "2": "Un traitement empirique à base de ceftriaxone intramusculaire et d'azithromycine orale en doses uniques sera mis en œuvre, dans l'attente des résultats microbiologiques.", "3": "Un antibiogramme doit être réalisé dans les cas où la gonorrhée est isolée.", "4": "2 à 3 semaines après le traitement, de nouveaux échantillons urétraux doivent être prélevés pour confirmer la guérison microbiologique.", "5": null}, "correct_option": 4, "explanations": {"1": {"exist": false, "char_ranges": [], "word_ranges": [], "text": ""}, "2": {"exist": false, "char_ranges": [], "word_ranges": [], "text": ""}, "3": {"exist": false, "char_ranges": [], "word_ranges": [], "text": ""}, "4": {"exist": true, "char_ranges": [[254, 482]], "word_ranges": [[38, 73]], "text": "toute personne atteinte de gonorrhée pharyngée et traitée avec un schéma alternatif doit revenir 14 jours après le traitement pour un test de guérison utilisant une culture ou un test d'amplification de l'acide nucléique (TAAN)."}, "5": {"exist": false, "char_ranges": [], "word_ranges": [], "text": ""}}}
{"id": 540, "year": 2021, "question_id_specific": 4, "full_question": "Un patient de 40 ans a commencé à ressentir une gêne dans la fesse droite après un long trajet en voiture. Depuis lors, il ne peut tolérer une position assise prolongée en raison de la récurrence de la douleur. Il ressent parfois un engourdissement du membre inférieur droit qui disparaît lorsqu'il se lève. L'imagerie par résonance magnétique et la radiographie de la hanche n'ont révélé aucune anomalie. L'examen de la sensibilité et des réflexes ostéotendineux ne montre aucune altération. Les manœuvres de rotation externe et d'abduction résistante, avec la hanche à 90 degrés de flexion, reproduisent la douleur. Le diagnostic le plus probable est le suivant :", "full_answer": "Le syndrome du piriforme est dû à la compression ou à l'empiètement du nerf sciatique par l'hypertrophie ou la contracture du muscle piriforme. Les symptômes caractéristiques du syndrome du piriforme comprennent une douleur fessière irradiant vers l'arrière de la cuisse et pouvant être accompagnée de paresthésie. Comme facteur déclenchant fréquent, le patient peut signaler qu'il s'est assis pendant une longue période sur une surface dure avant de ressentir la douleur. Le cas clinique présente la manœuvre PACE dans laquelle le patient résiste à la rotation externe et à l'abduction de la hanche jusqu'à 90° de flexion à l'examinateur. Il est considéré comme positif s'il reproduit les symptômes dans le contexte du syndrome pyramidal.", "type": "TRAUMATOLOGIE", "options": {"1": "Syndrome du conflit fémoro-acétabulaire.", "2": "Syndrome du tenseur du fascia latae.", "3": "Radiculopathie S1.", "4": "Syndrome pyramidal.", "5": null}, "correct_option": 4, "explanations": {"1": {"exist": false, "char_ranges": [], "word_ranges": [], "text": ""}, "2": {"exist": false, "char_ranges": [], "word_ranges": [], "text": ""}, "3": {"exist": false, "char_ranges": [], "word_ranges": [], "text": ""}, "4": {"exist": true, "char_ranges": [[473, 739]], "word_ranges": [[70, 113]], "text": "Le cas clinique présente la manœuvre PACE dans laquelle le patient résiste à la rotation externe et à l'abduction de la hanche jusqu'à 90° de flexion à l'examinateur. Il est considéré comme positif s'il reproduit les symptômes dans le contexte du syndrome pyramidal."}, "5": {"exist": false, "char_ranges": [], "word_ranges": [], "text": ""}}}
{"id": 266, "year": 2014, "question_id_specific": 138, "full_question": "Chez un patient atteint d'une bronchopneumopathie chronique obstructive (BPCO) légère en phase stable, on observe une polyglobulie et une insuffisance respiratoire. La radiographie du thorax ne montre pas d'altérations notables. Parmi les examens suivants, lequel envisagez-vous d'effectuer en premier lieu pour exclure la coexistence d'autres maladies pouvant justifier les résultats décrits ?", "full_answer": "L'approche est douteuse faute de données suffisantes. En termes de fréquence, la bonne réponse pourrait être la polygraphie respiratoire, surtout dans le cas d'un patient obèse, ce que nous ne savons pas, mais ce pourrait être aussi la bonne réponse 2 (échocardiogramme) en cas de suspicion de cardiopathie associée, ce qui est moins fréquent mais aussi possible. L'association de la BPCO au SAHOS est une cause fréquente de détérioration de la BPCO, supérieure à celle qui correspondrait à une altération fonctionnelle.", "type": "PNEUMOLOGIE", "options": {"1": "Polygraphie respiratoire.", "2": "Echocardiogramme.", "3": "Tomodensitométrie thoracique.", "4": "Scintigraphie pulmonaire.", "5": "Ponction de moelle osseuse."}, "correct_option": 1, "explanations": {"1": {"exist": true, "char_ranges": [[54, 137]], "word_ranges": [[7, 19]], "text": "En termes de fréquence, la bonne réponse pourrait être la polygraphie respiratoire,"}, "2": {"exist": true, "char_ranges": [[210, 363]], "word_ranges": [[33, 57]], "text": "ce pourrait être aussi la bonne réponse 2 (échocardiogramme) en cas de suspicion de cardiopathie associée, ce qui est moins fréquent mais aussi possible."}, "3": {"exist": false, "char_ranges": [], "word_ranges": [], "text": ""}, "4": {"exist": false, "char_ranges": [], "word_ranges": [], "text": ""}, "5": {"exist": false, "char_ranges": [], "word_ranges": [], "text": ""}}}
{"id": 13, "year": 2011, "question_id_specific": 150, "full_question": "Un garçon d'un an et demi s'est présenté aux urgences avec des douleurs abdominales et une jaunisse. À l'examen, une masse abdominale a été palpée. Une échographie a révélé la présence d'un kyste de la voie biliaire commune. Quelle approche thérapeutique devons-nous adopter ?", "full_answer": "Je pense que la bonne réponse est 4. Le kyste de la voie biliaire commune est une pathologie rare mais son traitement est chirurgical et consiste en une résection du kyste et une hépaticojéjunosotomie. La cholangiorésonance est nécessaire pour choisir la technique appropriée pour les quatre classes existantes de kyste de la voie biliaire commune (classification de Todani).", "type": "CHIRURGIE", "options": {"1": "Une ponction percutanée pour lavage péritonéal sera effectuée pour vérifier que le kyste contient de la bile afin de laisser un drain pour améliorer la douleur abdominale.", "2": "Une résection radicale de l'ensemble du canal biliaire est indiquée pour le remplacer par une anse intestinale.", "3": "Une laparotomie exploratoire et un drainage du kyste sont indiqués et lorsque la dilatation se résorbe, le drain est retiré.", "4": "Une scintigraphie par cholangiorésonance sera effectuée pour délimiter le kyste et une laparotomie sera indiquée pour la résection du kyste et l'anastomose du canal biliaire.", "5": "La scintigraphie HIDA est nécessaire pour délimiter le kyste en vue d'un drainage percutané sûr."}, "correct_option": 4, "explanations": {"1": {"exist": false, "char_ranges": [], "word_ranges": [], "text": ""}, "2": {"exist": false, "char_ranges": [], "word_ranges": [], "text": ""}, "3": {"exist": false, "char_ranges": [], "word_ranges": [], "text": ""}, "4": {"exist": true, "char_ranges": [[0, 375]], "word_ranges": [[0, 58]], "text": "Je pense que la bonne réponse est 4. Le kyste de la voie biliaire commune est une pathologie rare mais son traitement est chirurgical et consiste en une résection du kyste et une hépaticojéjunosotomie. La cholangiorésonance est nécessaire pour choisir la technique appropriée pour les quatre classes existantes de kyste de la voie biliaire commune (classification de Todani)."}, "5": {"exist": false, "char_ranges": [], "word_ranges": [], "text": ""}}}
{"id": 299, "year": 2016, "question_id_specific": 104, "full_question": "Un homme de 46 ans, consommateur de drogues injectables, s'est présenté au service des urgences avec de la fièvre accompagnée de frissons, une confusion mentale, une myalgie diffuse et une douleur intense à la main gauche depuis 24 heures, sans signes locaux clairs de phlogose. Il n'y avait pas d'antécédents de traumatisme. L'examen a révélé une température de 38,9°C, 120 battements par minute, une fréquence respiratoire de 30/min, une pression artérielle de 90/54 mm Hg. Les examens de laboratoire révèlent une leucocytose avec déplacement vers la gauche (25 000 leucocytes/mm3, 80 % de neutrophiles), une augmentation de la créatinine (1,6 mg/dL) et de la CK (138 U/L). Lequel des diagnostics suivants est le plus probable ?", "full_answer": "J'ai des doutes entre 1 et 2 ; exceptionnellement C. botulinum peut infecter des plaies traumatiques ou perforantes chez des UDVP utilisant de l'héroïne brune ou causer des sinusites chez des cocaïnomanes. En revanche, il peut s'agir d'une septicémie sévère dont la manifestation initiale est faible, avec rhabdomyolyse débutante et douleurs musculaires, dans ce cas il peut s'agir d'une fasciite nécrosante, où le germe le plus fréquemment en cause est S. pyogenes.", "type": "LES MALADIES INFECTIEUSES", "options": {"1": "Fasciite nécrosante streptococcique.", "2": "Gangrène due à Clostridium spp.", "3": "Cellulite mycobactérienne.", "4": "Erysipèle.", "5": null}, "correct_option": 1, "explanations": {"1": {"exist": true, "char_ranges": [[218, 408]], "word_ranges": [[33, 61]], "text": "revanche, il peut s'agir d'une septicémie sévère dont la manifestation initiale est faible, avec rhabdomyolyse débutante et douleurs musculaires, dans ce cas il peut s'agir d'une fasciite nécrosante,"}, "2": {"exist": true, "char_ranges": [[31, 205]], "word_ranges": [[8, 32]], "text": "exceptionnellement C. botulinum peut infecter des plaies traumatiques ou perforantes chez des UDVP utilisant de l'héroïne brune ou causer des sinusites chez des cocaïnomanes."}, "3": {"exist": false, "char_ranges": [], "word_ranges": [], "text": ""}, "4": {"exist": false, "char_ranges": [], "word_ranges": [], "text": ""}, "5": {"exist": false, "char_ranges": [], "word_ranges": [], "text": ""}}}
{"id": 16, "year": 2011, "question_id_specific": 131, "full_question": "Un homme de 62 ans, ayant des antécédents d'abus d'alcool, porteur du virus de l'hépatite C, traité par Ibuprofène pour une tendinite de l'épaule droite, consulte son dermatologue car après deux semaines de vacances à la plage, il constate l'apparition de vésicules tendues sur le dos de ses mains. A l'examen, il a également constaté une localisation et une légère hypertrichose malaire. Le diagnostic le plus probable est le suivant :", "full_answer": "Porphyrie cutanée tardive : 60% des patients atteints de PCT sont des hommes, beaucoup d'entre eux consomment de l'alcool en excès, et les femmes qui développent cette maladie sont généralement traitées avec des médicaments contenant des œstrogènes. La plupart sont des hommes présentant des signes de surcharge en fer, ce qui réduit l'activité de l'enzyme uroporphyrinogène décarboxylase, entraînant une élévation des uroporphyrines. Les infections par le VHC et le VIH ont été impliquées dans la précipitation de la PCT acquise. Il existe une forme héréditaire avec un schéma AD. Les patients atteints de PCT présentent des cloques sur la peau photoexposée, le plus souvent sur le dos des mains et le cuir chevelu. Outre la fragilité, ils peuvent présenter une hypertrichose, une hyperpigmentation, une alopécie cicatricielle et une induration sclérodermique.", "type": "DERMATOLOGIE", "options": {"1": "Epidermolyse bulleuse acquise.", "2": "Porphyrie cutanée tardive.", "3": "Réaction phototoxique.", "4": "Dermatite de contact.", "5": "Porphyrie aiguë intermittente."}, "correct_option": 2, "explanations": {"1": {"exist": false, "char_ranges": [], "word_ranges": [], "text": ""}, "2": {"exist": true, "char_ranges": [[0, 434]], "word_ranges": [[0, 62]], "text": "Porphyrie cutanée tardive : 60% des patients atteints de PCT sont des hommes, beaucoup d'entre eux consomment de l'alcool en excès, et les femmes qui développent cette maladie sont généralement traitées avec des médicaments contenant des œstrogènes. La plupart sont des hommes présentant des signes de surcharge en fer, ce qui réduit l'activité de l'enzyme uroporphyrinogène décarboxylase, entraînant une élévation des uroporphyrines."}, "3": {"exist": false, "char_ranges": [], "word_ranges": [], "text": ""}, "4": {"exist": false, "char_ranges": [], "word_ranges": [], "text": ""}, "5": {"exist": false, "char_ranges": [], "word_ranges": [], "text": ""}}}
{"id": 121, "year": 2012, "question_id_specific": 114, "full_question": "Femme de 47 ans. Originaire de Cochabamba (Bolivie), elle vit en Espagne depuis 8 ans. Au milieu de l'année 2009, elle a commencé à ressentir des palpitations, une dyspnée et des œdèmes progressifs dans les membres inférieurs. On lui a diagnostiqué une insuffisance cardiaque secondaire à une cardiomyopathie due à la maladie de Chagas. Quel est l'agent étiologique de cette maladie, qui est endémique dans plusieurs pays d'Amérique latine ?", "full_answer": "La bonne réponse est cinq. La maladie de Chagas est causée par Trypanosoma cruzi.", "type": "MICROBIOLOGIE", "options": {"1": "Toxoplasma gondii.", "2": "Trypanosoma brucei.", "3": "Lehismania donovani.", "4": "Giardia lamblia.", "5": "Trypanosoma cruzi."}, "correct_option": 5, "explanations": {"1": {"exist": false, "char_ranges": [], "word_ranges": [], "text": ""}, "2": {"exist": false, "char_ranges": [], "word_ranges": [], "text": ""}, "3": {"exist": false, "char_ranges": [], "word_ranges": [], "text": ""}, "4": {"exist": false, "char_ranges": [], "word_ranges": [], "text": ""}, "5": {"exist": true, "char_ranges": [[27, 81]], "word_ranges": [[5, 14]], "text": "La maladie de Chagas est causée par Trypanosoma cruzi."}}}
{"id": 241, "year": 2014, "question_id_specific": 229, "full_question": "Une femme obèse de 29 ans, sans antécédent particulier, consulte pour des céphalées intenses, bilatérales, pulsatiles, depuis un mois, accompagnées d'une diplopie horizontale et d'épisodes d'amaurose monoculaire durant quelques secondes. L'examen est normal, sauf la présence d'un œdème papillaire bilatéral. Lequel de ces examens vous permettra, selon vous, de confirmer pleinement le diagnostic ?", "full_answer": "Une fois de plus, nous sommes confrontés à un cas où il faut poser un diagnostic puis, dans ce cas, effectuer un test de confirmation. Le symptôme directeur est un œdème papillaire bilatéral qui indique une hypertension intracrânienne. Bien qu'une lésion occupant l'espace n'ait pas été exclue, compte tenu de l'examen neurologique normal et des autres symptômes, on peut déduire qu'il s'agit d'une hypertension intracrânienne idiopathique ou d'un pseudo-tumeur cérébrale. Les femmes en âge de procréer, l'obésité et la diplopie (due à l'atteinte du VIe nerf crânien) sont d'autres données évocatrices de cette entité. Il est caractéristique que la céphalée et le flou visuel s'aggravent soudainement le matin en se levant ou en changeant de position, produisant une cécité transitoire (Neurologie, Zarranz). La ponction lombaire montre invariablement une augmentation de la pression du liquide céphalo-rachidien, la bonne réponse est donc 4.", "type": "NEUROLOGIE", "options": {"1": "Échographie des troncs supra-aortiques.", "2": "Imagerie par résonance magnétique du crâne.", "3": "Électroencéphalogramme.", "4": "Ponction lombaire.", "5": "Potentiels évoqués visuels."}, "correct_option": 4, "explanations": {"1": {"exist": false, "char_ranges": [], "word_ranges": [], "text": ""}, "2": {"exist": false, "char_ranges": [], "word_ranges": [], "text": ""}, "3": {"exist": false, "char_ranges": [], "word_ranges": [], "text": ""}, "4": {"exist": true, "char_ranges": [[619, 942]], "word_ranges": [[94, 141]], "text": "Il est caractéristique que la céphalée et le flou visuel s'aggravent soudainement le matin en se levant ou en changeant de position, produisant une cécité transitoire (Neurologie, Zarranz). La ponction lombaire montre invariablement une augmentation de la pression du liquide céphalo-rachidien, la bonne réponse est donc 4."}, "5": {"exist": false, "char_ranges": [], "word_ranges": [], "text": ""}}}
{"id": 576, "year": 2022, "question_id_specific": 103, "full_question": "Un homme de 30 ans consulte pour des céphalées périoculaires unilatérales qui évoluent rapidement pour devenir très intenses. Les crises douloureuses durent 30 à 60 minutes, surviennent 2 à 3 fois par jour et s'accompagnent d'un larmoiement de l'œil douloureux et d'une rhinorrhée unilatérale. Quel est le traitement préventif le plus approprié ?", "full_answer": "Ils décrivent un cas caractéristique de céphalée en grappe. Elle est plus fréquente chez les hommes, la douleur est caractéristiquement périoculaire unilatérale, de grande intensité, de durée variable, généralement de quelques minutes et associée à des signes trigémino-autonomiques tels que rhinorrhée et injection conjonctivale unilatérale. Le traitement préventif par excellence est le vérapamil associé à un traitement descendant de corticostéroïdes, comme décrit dans la réponse 2.", "type": "NEUROLOGIE", "options": {"1": "Prednisone par voie orale avec arrêt au bout de 2 à 3 mois.", "2": "Prednisone orale avec sevrage en un mois, associée à du vérapamil oral à la dose de 80-180 mg toutes les 8 heures.", "3": "Carbamazépine à une dose de 600-1200 mg par jour.", "4": "Indométhacine à la dose de 50 mg toutes les 8 heures pendant un mois.", "5": null}, "correct_option": 2, "explanations": {"1": {"exist": false, "char_ranges": [], "word_ranges": [], "text": ""}, "2": {"exist": true, "char_ranges": [[0, 486]], "word_ranges": [[0, 66]], "text": "Ils décrivent un cas caractéristique de céphalée en grappe. Elle est plus fréquente chez les hommes, la douleur est caractéristiquement périoculaire unilatérale, de grande intensité, de durée variable, généralement de quelques minutes et associée à des signes trigémino-autonomiques tels que rhinorrhée et injection conjonctivale unilatérale. Le traitement préventif par excellence est le vérapamil associé à un traitement descendant de corticostéroïdes, comme décrit dans la réponse 2."}, "3": {"exist": false, "char_ranges": [], "word_ranges": [], "text": ""}, "4": {"exist": false, "char_ranges": [], "word_ranges": [], "text": ""}, "5": {"exist": false, "char_ranges": [], "word_ranges": [], "text": ""}}}
{"id": 504, "year": 2020, "question_id_specific": 87, "full_question": "Un trouble obsessionnel-compulsif (TOC) d'apparition soudaine chez un enfant de 9 ans présentant des tics et une chorée comme manifestations neurologiques doit être exclu :", "full_answer": "Nous présentons un cas de syndrome PANDAS, une complication rare d'une infection causée par un streptocoque bêta-hémolytique du groupe A.", "type": "PÉDIATRIE", "options": {"1": "Streptococcus viridans.", "2": "Streptocoque alpha-hémolytique.", "3": "Streptocoque bêta-hémolytique du groupe A.", "4": "Entérocoque.", "5": null}, "correct_option": 3, "explanations": {"1": {"exist": false, "char_ranges": [], "word_ranges": [], "text": ""}, "2": {"exist": false, "char_ranges": [], "word_ranges": [], "text": ""}, "3": {"exist": true, "char_ranges": [[0, 137]], "word_ranges": [[0, 20]], "text": "Nous présentons un cas de syndrome PANDAS, une complication rare d'une infection causée par un streptocoque bêta-hémolytique du groupe A."}, "4": {"exist": false, "char_ranges": [], "word_ranges": [], "text": ""}, "5": {"exist": false, "char_ranges": [], "word_ranges": [], "text": ""}}}
{"id": 367, "year": 2016, "question_id_specific": 119, "full_question": "Une femme de 38 ans, non-fumeuse, chez qui on avait précédemment diagnostiqué une rhinite atopique, présente depuis deux mois une toux intermittente et une respiration sifflante, principalement la nuit. La radiographie pulmonaire est normale. Spirométrie dans les limites de la normale avec test bronchodilatateur négatif. Lequel des examens suivants demanderez-vous ensuite ?", "full_answer": "Une spirométrie normale et un test au bronchodilatateur ponctuellement négatif n'excluent pas l'asthme, et le tableau clinique du patient est très évocateur d'asthme, de sorte qu'un test à la méthacholine confirmerait le diagnostic, au moins d'hyperréactivité bronchique.", "type": "PNEUMOLOGIE ET CHIRURGIE THORACIQUE", "options": {"1": "Radiographie des sinus paranasaux.", "2": "CT thoracique.", "3": "Test à la méthacholine.", "4": "Prick-test.", "5": null}, "correct_option": 3, "explanations": {"1": {"exist": false, "char_ranges": [], "word_ranges": [], "text": ""}, "2": {"exist": false, "char_ranges": [], "word_ranges": [], "text": ""}, "3": {"exist": true, "char_ranges": [[0, 271]], "word_ranges": [[0, 37]], "text": "Une spirométrie normale et un test au bronchodilatateur ponctuellement négatif n'excluent pas l'asthme, et le tableau clinique du patient est très évocateur d'asthme, de sorte qu'un test à la méthacholine confirmerait le diagnostic, au moins d'hyperréactivité bronchique."}, "4": {"exist": false, "char_ranges": [], "word_ranges": [], "text": ""}, "5": {"exist": false, "char_ranges": [], "word_ranges": [], "text": ""}}}
{"id": 444, "year": 2018, "question_id_specific": 104, "full_question": "Une femme de 36 ans se présente à la clinique après avoir fait trois fausses couches au cours du premier trimestre. Elle n'a jamais eu de grossesse à terme. Les tests à demander en premier lieu ne comprennent PAS :", "full_answer": "La bonne réponse est 1. En effet, l'hystérosalpingographie permet de déterminer s'il y a une obstruction des trompes et, par conséquent, s'il n'y a pas de fécondation. Ce n'est pas le cas de notre patiente, où la fécondation a lieu et où la grossesse est même implantée. Les autres tests diagnostiques seraient inclus.", "type": "GYNÉCOLOGIE ET OBSTÉTRIQUE", "options": {"1": "Hystérosalpingographie.", "2": "Caryotype du sang périphérique des deux partenaires.", "3": "Détermination des anticorps antiphospholipides.", "4": "Échographie vaginale.", "5": null}, "correct_option": 1, "explanations": {"1": {"exist": true, "char_ranges": [[0, 270]], "word_ranges": [[0, 47]], "text": "La bonne réponse est 1. En effet, l'hystérosalpingographie permet de déterminer s'il y a une obstruction des trompes et, par conséquent, s'il n'y a pas de fécondation. Ce n'est pas le cas de notre patiente, où la fécondation a lieu et où la grossesse est même implantée."}, "2": {"exist": true, "char_ranges": [[271, 318]], "word_ranges": [[47, 53]], "text": "Les autres tests diagnostiques seraient inclus."}, "3": {"exist": true, "char_ranges": [[271, 318]], "word_ranges": [[47, 53]], "text": "Les autres tests diagnostiques seraient inclus."}, "4": {"exist": true, "char_ranges": [[271, 318]], "word_ranges": [[47, 53]], "text": "Les autres tests diagnostiques seraient inclus."}, "5": {"exist": false, "char_ranges": [], "word_ranges": [], "text": ""}}}
{"id": 171, "year": 2013, "question_id_specific": 226, "full_question": "Un patient de 26 ans, athlète de compétition, a été étudié en raison du décès soudain de son frère. Les examens complémentaires détectent l'existence d'une cardiomyopathie hypertrophique obstructive. Cochez l'option FAUX en ce qui concerne cette pathologie :", "full_answer": "Le myocarde est plus épais qu'il ne devrait l'être, principalement dans le septum, et il est si épais qu'il obstrue l'écoulement du ventricule gauche, produisant une sténose \"fonctionnelle\" avec un gradient de pression. Notre objectif sera tout d'abord de \"détendre\" le myocarde : à cette fin, un bêta-bloquant est recommandé, tandis que la digoxine, qui est un bon inotrope positif, augmente le gradient et est contre-indiquée. Le sport n'est pas très recommandé, étant donné le risque accru de mort subite associé à cette pathologie. Et si nous avons un septum adipeux, qui fait saillie dans la cavité ventriculaire gauche, la déplétion volumique n'est pas conseillée, car nous augmenterions également le gradient. Quant au point 5, il est vrai qu'il est conseillé d'effectuer un examen minutieux et un ECG chez les parents de patients souffrant d'hypertrophie.", "type": "CARDIOLOGIE ET CHIRURGIE CARDIOVASCULAIRE", "options": {"1": "Le traitement par bêta-bloquants permet de réduire les symptômes.", "2": "L'abandon du sport de compétition devrait être recommandé.", "3": "La digoxine est utile pour réduire le gradient sous-aortique.", "4": "Les diurétiques doivent être évités ou utilisés avec beaucoup de précautions.", "5": "Compte tenu de la nature héréditaire de la maladie, un dépistage devrait être effectué chez les autres membres de la famille directe."}, "correct_option": 3, "explanations": {"1": {"exist": true, "char_ranges": [[220, 326]], "word_ranges": [[33, 50]], "text": "Notre objectif sera tout d'abord de \"détendre\" le myocarde : à cette fin, un bêta-bloquant est recommandé,"}, "2": {"exist": true, "char_ranges": [[429, 535]], "word_ranges": [[66, 84]], "text": "Le sport n'est pas très recommandé, étant donné le risque accru de mort subite associé à cette pathologie."}, "3": {"exist": true, "char_ranges": [[327, 428]], "word_ranges": [[50, 66]], "text": "tandis que la digoxine, qui est un bon inotrope positif, augmente le gradient et est contre-indiquée."}, "4": {"exist": true, "char_ranges": [[536, 716]], "word_ranges": [[84, 111]], "text": "Et si nous avons un septum adipeux, qui fait saillie dans la cavité ventriculaire gauche, la déplétion volumique n'est pas conseillée, car nous augmenterions également le gradient."}, "5": {"exist": true, "char_ranges": [[717, 863]], "word_ranges": [[111, 135]], "text": "Quant au point 5, il est vrai qu'il est conseillé d'effectuer un examen minutieux et un ECG chez les parents de patients souffrant d'hypertrophie."}}}
{"id": 555, "year": 2022, "question_id_specific": 51, "full_question": "Une femme de 55 ans, atteinte d'un diabète de type 1 diagnostiqué à l'âge de 15 ans, se plaint de nausées et de ballonnements, surtout après les repas. L'évaluation est compatible avec une gastroparésie diabétique. Lequel des médicaments suivants serait le traitement le plus approprié pour améliorer ses symptômes ?", "full_answer": "Il a été approuvé comme traitement de la gastroparésie par la Food and Drug Administration (FDA) des États-Unis en 1979 et reste le médicament de première intention. Il agit comme un prokinétique par son effet antagoniste sur le récepteur de la dopamine 2 (D2) qui favorise la vidange gastrique et, en outre, se lie au récepteur de la 5-hydroxytryptamine 4 (sérotonine 5-HT4) pour stimuler les voies nerveuses cholinergiques dans l'estomac. Physiologiquement, il accélère le transit intestinal en augmentant le tonus et l'amplitude des contractions gastriques, augmente la pression du sphincter inférieur de l'œsophage et améliore la coordination antropyloroduodénale. En outre, cet agent antiémétique procure un soulagement par l'antagonisme des récepteurs dopaminergiques centraux et périphériques.", "type": "ENDOCRINOLOGIE", "options": {"1": "Lansoprazole.", "2": "Ondansétron.", "3": "Lopéramide.", "4": "Métoclopramide.", "5": null}, "correct_option": 4, "explanations": {"1": {"exist": false, "char_ranges": [], "word_ranges": [], "text": ""}, "2": {"exist": false, "char_ranges": [], "word_ranges": [], "text": ""}, "3": {"exist": false, "char_ranges": [], "word_ranges": [], "text": ""}, "4": {"exist": true, "char_ranges": [[0, 165]], "word_ranges": [[0, 27]], "text": "Il a été approuvé comme traitement de la gastroparésie par la Food and Drug Administration (FDA) des États-Unis en 1979 et reste le médicament de première intention."}, "5": {"exist": false, "char_ranges": [], "word_ranges": [], "text": ""}}}
{"id": 320, "year": 2016, "question_id_specific": 142, "full_question": "Un homme de 40 ans, sans antécédents particuliers, s'est présenté avec une arthrite aiguë du genou droit évoluant depuis 5 jours et une fièvre atteignant 37,7°C. Il n'a pas signalé de traumatisme. Il n'a signalé aucun antécédent de traumatisme. Il a présenté un épanchement articulaire, une chaleur à la palpation et une limitation fonctionnelle due à la douleur. Une arthrocentèse diagnostique a été effectuée, montrant 30000 cellules/microlitre avec une prédominance de neutrophiles. L'examen en lumière polarisée a révélé des cristaux rhomboïdaux et cylindriques avec une biréfringence faiblement positive. La radiographie du genou révèle des dépôts radiodenses dans le ménisque externe du genou. Cochez la réponse FAUX :", "full_answer": "Il s'agit d'une chondrocalcinose. Il n'existe pas de traitement pour cette maladie, seulement un traitement des poussées (colchicine, AINS ou corticoïdes), bien que parfois, si les poussées sont régulières, la colchicine soit laissée en place. L'allopurinol n'a pas fait la preuve de son efficacité dans cette arthropathie microcristalline, mais il a fait la preuve de son efficacité dans la goutte.", "type": "RHEUMATOLOGIE", "options": {"1": "Le traitement initial consiste à administrer des anti-inflammatoires non stéroïdiens et à aspirer le liquide synovial.", "2": "Une culture du liquide synovial doit être effectuée pour exclure une synovite infectieuse.", "3": "Une fois la poussée aiguë passée, un traitement à l'allopurinol doit être instauré pour réduire l'incidence des futures poussées d'arthrite.", "4": "Le diagnostic le plus probable est celui d'une arthrite aiguë due à un dépôt de cristaux de pyrophosphate de calcium (pseudogoutte).", "5": null}, "correct_option": 3, "explanations": {"1": {"exist": true, "char_ranges": [[34, 154]], "word_ranges": [[4, 21]], "text": "Il n'existe pas de traitement pour cette maladie, seulement un traitement des poussées (colchicine, AINS ou corticoïdes),"}, "2": {"exist": false, "char_ranges": [], "word_ranges": [], "text": ""}, "3": {"exist": true, "char_ranges": [[244, 399]], "word_ranges": [[35, 60]], "text": "L'allopurinol n'a pas fait la preuve de son efficacité dans cette arthropathie microcristalline, mais il a fait la preuve de son efficacité dans la goutte."}, "4": {"exist": true, "char_ranges": [[244, 399]], "word_ranges": [[35, 60]], "text": "L'allopurinol n'a pas fait la preuve de son efficacité dans cette arthropathie microcristalline, mais il a fait la preuve de son efficacité dans la goutte."}, "5": {"exist": false, "char_ranges": [], "word_ranges": [], "text": ""}}}
{"id": 617, "year": 2022, "question_id_specific": 119, "full_question": "Femme de 13 ans, sans antécédents pertinents, avec ménarche depuis 3 mois, suivie depuis l'âge de 10 ans pour une scoliose idiopathique qui s'est aggravée. À l'examen physique, elle présentait une bosse de 7 degrés au test d'Adams et, à la scoliographie, une courbe thoraco-lombaire T4-L1 de 35 degrés de Cobb et une courbe de Risser de 0 :", "full_answer": "On nous présente le cas d'une patiente de 13 ans, avec une ménarche datant de 3 mois, une courbure thoraco-lombaire T4-L1 de 35º et un Risser 0. En règle générale, on peut dire qu'en dessous de 50º, le traitement est habituellement conservateur, avec deux options : l'observation ou l'utilisation d'un appareil orthopédique pour ralentir l'évolution. Dans ce cas, compte tenu de la croissance restante (ménarche il y a 3 mois et Risser 0), l'option la plus indiquée serait l'utilisation d'une orthèse (Réponse 2 correcte).", "type": "TRAUMATOLOGIE", "options": {"1": "Recommander la natation et réexaminer la situation dans trois mois.", "2": "Prescrire une orthèse.", "3": "Consulter un physiothérapeute pour l'élasticité de la colonne vertébrale.", "4": "Révision dans 6 mois avec une nouvelle radiographie.", "5": null}, "correct_option": 2, "explanations": {"1": {"exist": false, "char_ranges": [], "word_ranges": [], "text": ""}, "2": {"exist": true, "char_ranges": [[0, 522]], "word_ranges": [[0, 84]], "text": "On nous présente le cas d'une patiente de 13 ans, avec une ménarche datant de 3 mois, une courbure thoraco-lombaire T4-L1 de 35º et un Risser 0. En règle générale, on peut dire qu'en dessous de 50º, le traitement est habituellement conservateur, avec deux options : l'observation ou l'utilisation d'un appareil orthopédique pour ralentir l'évolution. Dans ce cas, compte tenu de la croissance restante (ménarche il y a 3 mois et Risser 0), l'option la plus indiquée serait l'utilisation d'une orthèse (Réponse 2 correcte)."}, "3": {"exist": false, "char_ranges": [], "word_ranges": [], "text": ""}, "4": {"exist": false, "char_ranges": [], "word_ranges": [], "text": ""}, "5": {"exist": false, "char_ranges": [], "word_ranges": [], "text": ""}}}
{"id": 146, "year": 2012, "question_id_specific": 231, "full_question": "Un nourrisson de 7 mois a été adressé au service des urgences par son pédiatre local pour une suspicion de maladie de Kawasaki. Il a fait état d'une fièvre atteignant 39,5 °C depuis 5 jours, qui n'avait pas diminué malgré un traitement à l'amoxicilline 72 heures plus tôt pour une pharyngotonsillite aiguë présumée. Au cours des dernières 24 heures, il a présenté une éruption érythémateuse et prurigineuse d'évolution céphalocaudale et, au cours des derniers jours, ses parents ont remarqué une toux et une conjonctivite intense. Aucune rhinorrhée n'a été signalée. L'examen clinique a montré des bruits sous-crépitants dans les deux bases pulmonaires sans tachypnée ni signes de détresse respiratoire. L'examen oropharyngé a révélé des taches blanchâtres sur la muqueuse jugale. Quel est le diagnostic le plus probable ?", "full_answer": "La bonne réponse est 5. L'indice réside dans les lésions blanchâtres de la muqueuse jugale correspondant aux taches de Koplik, qui sont pathognomoniques de la rougeole, bien qu'à titre d'observation je dirais qu'elles apparaissent généralement dans la phase prodromique de la maladie et ne sont souvent plus visibles lorsque l'exanthème apparaît. Dans ce cas, vous décrivez une rougeole compliquée par une pneumonie. C'est une question très actuelle.", "type": "PÉDIATRIE", "options": {"1": "Maladie de Kawasaki.", "2": "Scarlatine.", "3": "Réaction allergique à l'amoxicilline.", "4": "Mononucléose infectieuse.", "5": "La rougeole."}, "correct_option": 5, "explanations": {"1": {"exist": false, "char_ranges": [], "word_ranges": [], "text": ""}, "2": {"exist": false, "char_ranges": [], "word_ranges": [], "text": ""}, "3": {"exist": false, "char_ranges": [], "word_ranges": [], "text": ""}, "4": {"exist": false, "char_ranges": [], "word_ranges": [], "text": ""}, "5": {"exist": true, "char_ranges": [[24, 168]], "word_ranges": [[5, 26]], "text": "L'indice réside dans les lésions blanchâtres de la muqueuse jugale correspondant aux taches de Koplik, qui sont pathognomoniques de la rougeole,"}}}
{"id": 467, "year": 2020, "question_id_specific": 92, "full_question": "Une femme qui a des difficultés à marcher après une intervention chirurgicale gynécologique. Elle ressent une légère douleur dans la cuisse et sa jambe ne parvient pas à se reposer. À l'examen, elle présente une faiblesse de la flexion de la hanche et de l'extension du genou, ainsi qu'une dysesthésie de la face antérieure de la cuisse. Quel est le diagnostic le plus probable ?", "full_answer": "Le nerf fémoral (bien que rare) peut être endommagé lors d'une césarienne, d'une hystérectomie ou d'une opération du bas-ventre. Cette lésion du nerf provoque une hypoesthésie et une faiblesse le long de sa distribution. La fémoroneuropathie se manifeste par une paralysie (25 % des cas) des muscles quadriceps, une abolition des réflexes rotuliens et une hypoesthésie de la partie antérieure et interne de la cuisse. La réponse 1 est donc correcte. Le nerf cutané fémoral ou nerf cutané fémoral latéral (L2,L3) n'est que sensitif et est responsable de la sensation de la peau de la face latérale et antérieure de la cuisse jusqu'au genou, il n'expliquerait donc pas le problème moteur de la marche et 2 est faux. Le nerf obturateur (L2-L4) est un nerf mixte. Au niveau moteur, il implique l'obturateur externe, le pectiné, l'approximateur court, les approximateurs (adducteurs) et le gracile. Au niveau sensitif, il concerne la capsule de l'articulation coxo-fémorale, la face médiale de la cuisse au-dessus du genou. Elle n'affecte donc ni la sensation de la face antérieure de la cuisse ni l'extension du genou, donc 3 est faux. Une lésion du nerf sciatique au niveau moteur affecte la flexion du genou, pas son extension, et au niveau distal du genou (il n'est pas fait mention de symptômes distaux) et au niveau sensoriel, elle est également distale de la face externe de la jambe et du pied, de sorte que le point 4 est également faux.", "type": "CHIRURGIE ORTHOPÉDIQUE ET TRAUMATOLOGIE", "options": {"1": "Neuropathie du nerf fémoral.", "2": "Meralgia paraesthetica du nerf cutané fémoral.", "3": "Neuropathie du nerf obturateur.", "4": "Neuropathie du nerf sciatique.", "5": null}, "correct_option": 1, "explanations": {"1": {"exist": true, "char_ranges": [[0, 449]], "word_ranges": [[0, 71]], "text": "Le nerf fémoral (bien que rare) peut être endommagé lors d'une césarienne, d'une hystérectomie ou d'une opération du bas-ventre. Cette lésion du nerf provoque une hypoesthésie et une faiblesse le long de sa distribution. La fémoroneuropathie se manifeste par une paralysie (25 % des cas) des muscles quadriceps, une abolition des réflexes rotuliens et une hypoesthésie de la partie antérieure et interne de la cuisse. La réponse 1 est donc correcte."}, "2": {"exist": true, "char_ranges": [[450, 713]], "word_ranges": [[71, 118]], "text": "Le nerf cutané fémoral ou nerf cutané fémoral latéral (L2,L3) n'est que sensitif et est responsable de la sensation de la peau de la face latérale et antérieure de la cuisse jusqu'au genou, il n'expliquerait donc pas le problème moteur de la marche et 2 est faux."}, "3": {"exist": true, "char_ranges": [[714, 1131]], "word_ranges": [[118, 183]], "text": "Le nerf obturateur (L2-L4) est un nerf mixte. Au niveau moteur, il implique l'obturateur externe, le pectiné, l'approximateur court, les approximateurs (adducteurs) et le gracile. Au niveau sensitif, il concerne la capsule de l'articulation coxo-fémorale, la face médiale de la cuisse au-dessus du genou. Elle n'affecte donc ni la sensation de la face antérieure de la cuisse ni l'extension du genou, donc 3 est faux."}, "4": {"exist": true, "char_ranges": [[1132, 1441]], "word_ranges": [[183, 240]], "text": "Une lésion du nerf sciatique au niveau moteur affecte la flexion du genou, pas son extension, et au niveau distal du genou (il n'est pas fait mention de symptômes distaux) et au niveau sensoriel, elle est également distale de la face externe de la jambe et du pied, de sorte que le point 4 est également faux."}, "5": {"exist": false, "char_ranges": [], "word_ranges": [], "text": ""}}}
{"id": 423, "year": 2018, "question_id_specific": 89, "full_question": "Une femme de 55 ans, obèse, diabétique de type 2, sous metformine, avec un taux d'Hb A1c de 8 %, a des antécédents d'infections urinaires récurrentes. Parmi les options thérapeutiques suivantes à associer à la metformine, laquelle vous semble la MOINS appropriée ?", "full_answer": "Les effets secondaires les plus courants des inhibiteurs du SGLT2 sont les infections des voies génito-urinaires. Étant donné que notre patiente souffre d'infections urinaires récurrentes, le médicament le moins indiqué pour le contrôle de son diabète serait l'option 4.", "type": "ENDOCRINOLOGIE", "options": {"1": "Inhibiteurs de la DPP4.", "2": "Analogues du GLP1.", "3": "Insuline basale.", "4": "Inhibiteurs du SGLT2.", "5": null}, "correct_option": 4, "explanations": {"1": {"exist": false, "char_ranges": [], "word_ranges": [], "text": ""}, "2": {"exist": false, "char_ranges": [], "word_ranges": [], "text": ""}, "3": {"exist": false, "char_ranges": [], "word_ranges": [], "text": ""}, "4": {"exist": true, "char_ranges": [[0, 270]], "word_ranges": [[0, 39]], "text": "Les effets secondaires les plus courants des inhibiteurs du SGLT2 sont les infections des voies génito-urinaires. Étant donné que notre patiente souffre d'infections urinaires récurrentes, le médicament le moins indiqué pour le contrôle de son diabète serait l'option 4."}, "5": {"exist": false, "char_ranges": [], "word_ranges": [], "text": ""}}}
{"id": 449, "year": 2018, "question_id_specific": 151, "full_question": "Un garçon de 13 ans présente des céphalées subaiguës et une diplopie. L'examen neurologique révèle une paralysie du regard vertical et l'IRM montre une lésion prenant le contraste dans la région pinéale, obstruant l'aqueduc de Sylvius. Le diagnostic le plus probable est le suivant :", "full_answer": "Les tumeurs germinales sont les tumeurs les plus fréquentes chez les enfants dans la région pinéale. Elles ont tendance à se capitonner de manière homogène et provoquent une hydrocéphalie triventriculaire et une hydrocéphalie de Parinaud. Les glioblastomes surviennent généralement chez les adultes au niveau lobaire, les médulloblastomes sont les tumeurs malignes les plus fréquentes au niveau de la fosse postérieure chez les enfants (vermis et toit du IVe ventricule) mais rares au niveau de la pinéale et les méningiomes sont rares chez les enfants et également dans la région de la pinéale.", "type": "NEUROSURGÉRIE", "options": {"1": "Glioblastome.", "2": "Médulloblastome.", "3": "Tumeur des cellules germinales.", "4": "Méningiome.", "5": null}, "correct_option": 3, "explanations": {"1": {"exist": true, "char_ranges": [[239, 317]], "word_ranges": [[35, 45]], "text": "Les glioblastomes surviennent généralement chez les adultes au niveau lobaire,"}, "2": {"exist": true, "char_ranges": [[318, 505]], "word_ranges": [[45, 76]], "text": "les médulloblastomes sont les tumeurs malignes les plus fréquentes au niveau de la fosse postérieure chez les enfants (vermis et toit du IVe ventricule) mais rares au niveau de la pinéale"}, "3": {"exist": true, "char_ranges": [[0, 238]], "word_ranges": [[0, 35]], "text": "Les tumeurs germinales sont les tumeurs les plus fréquentes chez les enfants dans la région pinéale. Elles ont tendance à se capitonner de manière homogène et provoquent une hydrocéphalie triventriculaire et une hydrocéphalie de Parinaud."}, "4": {"exist": true, "char_ranges": [[509, 595]], "word_ranges": [[77, 92]], "text": "les méningiomes sont rares chez les enfants et également dans la région de la pinéale."}, "5": {"exist": false, "char_ranges": [], "word_ranges": [], "text": ""}}}
{"id": 433, "year": 2018, "question_id_specific": 118, "full_question": "Luis est un jeune homme de 25 ans qui a subi une splénectomie après un accident de vélo il y a un an. Il a un chien qui l'a mordu il y a 24 heures et qui lui a causé une petite blessure à la main droite. Elle s'est rendue à son centre de santé (situé à 3 heures de l'hôpital le plus proche) avec une fièvre de 39ºC, une douleur au niveau de la plaie et un malaise général. A l'examen, la tension artérielle est de 100/60 mm Hg, la fréquence cardiaque est de 110 battements par minute, la plaie est légèrement enflée mais sans pus. Laquelle des mesures suivantes est la plus indiquée à ce stade ?", "full_answer": "Cette question a été compliquée pour nous. La question ne contient pas toutes les informations nécessaires, mais avec ce que nous avons, nous pouvons essayer d'y répondre. Le chien est domestique, connu et peut être surveillé, nous devrions donc attendre que la période d'observation soit passée et aucune prophylaxie n'est recommandée. À ce stade, nous ne devrons nous préoccuper que de l'éventuelle bactériémie qui pourrait survenir chez un patient présentant un risque élevé en raison d'une asplénie. Le traitement de choix pour les morsures de chien est l'amoxicilline/acide clavulanique, mais il n'est pas disponible. Nous pouvons donc utiliser un autre traitement comme la clindamycine plus une céphalosporine ou la clindamycine + la moxifloxacine, mais il n'est pas inclus dans les alternatives. Dans une première tentative, nous avons répondu 4, mais la bonne réponse est 3, en raison de la deuxième partie de la question ; il s'agit d'un patient à haut risque avec des signes précoces de septicémie et il devrait donc être envoyé à l'hôpital, ici la clé plus que dans l'antibiotique est dans la position à prendre, dans la deuxième partie de chaque réponse. Il pourrait être contesté car il n'est pas précisé dans quelle communauté la morsure a eu lieu (si c'était à Ceuta ou Melilla, le protocole peut varier).", "type": "MALADIES INFECTIEUSES ET MICROBIOLOGIE", "options": {"1": "Envoyer à l'hôpital pour la vaccination contre la rage et le tétanos et garder en observation.", "2": "Nettoyer la plaie et administrer des gammaglobulines non spécifiques par voie intramusculaire.", "3": "Administrer 400 mg de moxifloxacine par voie orale et envoyer le patient à l'hôpital.", "4": "Administrer 600 mg de clindamycine par voie orale toutes les 8 heures et observer.", "5": null}, "correct_option": 3, "explanations": {"1": {"exist": false, "char_ranges": [], "word_ranges": [], "text": ""}, "2": {"exist": false, "char_ranges": [], "word_ranges": [], "text": ""}, "3": {"exist": true, "char_ranges": [[803, 1166]], "word_ranges": [[121, 186]], "text": "Dans une première tentative, nous avons répondu 4, mais la bonne réponse est 3, en raison de la deuxième partie de la question ; il s'agit d'un patient à haut risque avec des signes précoces de septicémie et il devrait donc être envoyé à l'hôpital, ici la clé plus que dans l'antibiotique est dans la position à prendre, dans la deuxième partie de chaque réponse."}, "4": {"exist": true, "char_ranges": [[803, 1166]], "word_ranges": [[121, 186]], "text": "Dans une première tentative, nous avons répondu 4, mais la bonne réponse est 3, en raison de la deuxième partie de la question ; il s'agit d'un patient à haut risque avec des signes précoces de septicémie et il devrait donc être envoyé à l'hôpital, ici la clé plus que dans l'antibiotique est dans la position à prendre, dans la deuxième partie de chaque réponse."}, "5": {"exist": false, "char_ranges": [], "word_ranges": [], "text": ""}}}
{"id": 337, "year": 2016, "question_id_specific": 31, "full_question": "Une femme de 20 ans présentant une tumeur ovarienne kystique solide de 15 cm détectée par échographie après avoir présenté des symptômes abdominaux non spécifiques. Lors de l'étude histopathologique de l'échantillon correspondant, des dents, des poils, des zones d'épithélium intestinal, des zones d'épithélium malpighien (15 %) et d'épithélium bronchique, ainsi que des éléments neuroectodermiques et embryonnaires ont été trouvés dans plusieurs des préparations histologiques. En ce qui concerne ce cas, veuillez indiquer le bon diagnostic :", "full_answer": "La réponse est 3, tératome kystique mature. Ceci est dû aux caractéristiques spécifiques de la tumeur : les tératomes matures sont constitués de tissus adultes, issus des trois feuillets embryonnaires, avec une nette prédominance des éléments ectodermiques (épiderme, structures pilo-sébacées et tissu neural). Le tératome mature est une tumeur des cellules germinales ovariennes.", "type": "GYNÉCOLOGIE ET OBSTÉTRIQUE", "options": {"1": "Tératocarcinome.", "2": "Tératome immature.", "3": "Tératome kystique mature.", "4": "Dysgerminome.", "5": null}, "correct_option": 3, "explanations": {"1": {"exist": false, "char_ranges": [], "word_ranges": [], "text": ""}, "2": {"exist": false, "char_ranges": [], "word_ranges": [], "text": ""}, "3": {"exist": true, "char_ranges": [[44, 310]], "word_ranges": [[7, 43]], "text": "Ceci est dû aux caractéristiques spécifiques de la tumeur : les tératomes matures sont constitués de tissus adultes, issus des trois feuillets embryonnaires, avec une nette prédominance des éléments ectodermiques (épiderme, structures pilo-sébacées et tissu neural)."}, "4": {"exist": false, "char_ranges": [], "word_ranges": [], "text": ""}, "5": {"exist": false, "char_ranges": [], "word_ranges": [], "text": ""}}}
{"id": 296, "year": 2016, "question_id_specific": 17, "full_question": "Un homme de 47 ans, fumeur actif, ayant des antécédents de toxicomanie par injection, s'est présenté au service des urgences deux heures après avoir ressenti une douleur pleurétique aiguë à la côte gauche et une dyspnée progressive jusqu'à ce qu'il se repose. Au cours des deux semaines précédentes, il avait présenté de la fièvre, un malaise général et une toux sèche. À l'examen, le patient semble sévère, tachypnéique à 36 tours/minute, avec un tirage sus-claviculaire, une saturation en oxygène par oxymétrie de pouls de 81 %, une cyanose, des sueurs, une tachycardie à 135 battements par minute et une tension artérielle de 75/47 mm Hg. La trachée est déviée vers le côté droit et l'auscultation montre une absence totale de murmure vésiculaire dans l'hémithorax gauche. La sérologie VIH est positive avec une numération lymphocytaire ADC de 176 cellules/ml. Quel est le diagnostic le plus probable ?", "full_answer": "Patient présentant un facteur de risque d'acquisition du VIH, venu aux urgences pour un début subaigu de 2 semaines d'évolution (fièvre, malaise et toux sèche) qui s'est aggravé brutalement (douleur pleurétique) et progressivement (dyspnée). L'examen physique suggère un pneumothorax (tachypnée, tiraillement, faible saturation, cyanose, diaphorèse, tachycardie, trachée déviée et absence de murmure vésiculaire). Il présente une sérologie VIH positive et un taux de CD4 inférieur à 200 cellules/ml. La radiographie montre une hyperclarté dans le champ pulmonaire gauche avec absence de vaisseaux entre le bord du poumon et la paroi thoracique du côté gauche, un déplacement du médiastin et de la trachée vers le côté droit, et un moignon dans le hile pulmonaire gauche correspondant au poumon affaissé. La suspicion de VIH est élevée et bien qu'elle soit en attente de confirmation, dans l'urgence, il est valable d'assumer cette option. Cela dit, je cherche des pathologies associées à cette image, la rupture d'une bulle bien qu'elle soit liée à l'apparition de symptômes soudains ne va pas de pair avec les autres symptômes (nous devons chercher quelque chose qui explique tout ou presque), la pneumonie bien qu'il soit vrai qu'elle puisse être liée à de nombreux symptômes n'est pas en corrélation avec l'image, La tuberculose pulmonaire pourrait se manifester par tous les symptômes ci-dessus et provoquer un pneumothorax, mais ici, chez un patient présentant un tableau respiratoire évoluant depuis plus de 7 jours et se compliquant brusquement, chez un patient présentant une forte suspicion d'infection par le VIH, l'option la plus claire est la pneumonie à P. jirovecii. Il s'agit d'une pneumonie à P. jirovecii. Chez 10% des patients atteints de Pneumocystis, des pneumatocèles apparaissent, qui peuvent parfois s'ouvrir dans l'espace pleural en produisant un pneumothorax spontané. Pour rappel, les germes les plus fréquents dans les atteintes pulmonaires des patients infectés par le VIH/SIDA sont P. jiroveci (seul ou associé à la tuberculose ou à d'autres mycobactéries), S. pneumoniae, H. influenzae, S. aureus, cytomégalovirus, Legionella, staphylocoques et bacilles à Gram négatif (Pseudomonas). Les patients atteints de pneumonie à P. jirovecii ont un taux de CD4 inférieur à 250 cellules/mL dans 90 % des cas. Les autres agents susceptibles de provoquer une atteinte pulmonaire sont moins fréquemment Rhodococcus equi, Nocardia, Aspergillus, Cryptococcus, Toxoplasma et Leishmania. Bien que la fréquence des différentes infections opportunistes ait varié en raison de l'administration de traitements antirétroviraux hautement efficaces, notre patient se trouve dans un contexte où il n'a pas encore reçu de traitement antirétroviral.", "type": "LES MALADIES INFECTIEUSES", "options": {"1": "Rupture de bulle emphysémateuse sous-pleurale.", "2": "Pneumonie bactérienne nécrosante d'origine communautaire.", "3": "Tuberculose pulmonaire.", "4": "Pneumonie à Pneumocystis jiroveci.", "5": null}, "correct_option": 4, "explanations": {"1": {"exist": true, "char_ranges": [[1001, 1194]], "word_ranges": [[150, 182]], "text": "la rupture d'une bulle bien qu'elle soit liée à l'apparition de symptômes soudains ne va pas de pair avec les autres symptômes (nous devons chercher quelque chose qui explique tout ou presque),"}, "2": {"exist": true, "char_ranges": [[1195, 1316]], "word_ranges": [[182, 202]], "text": "la pneumonie bien qu'il soit vrai qu'elle puisse être liée à de nombreux symptômes n'est pas en corrélation avec l'image,"}, "3": {"exist": true, "char_ranges": [[1466, 1680]], "word_ranges": [[223, 257]], "text": "un tableau respiratoire évoluant depuis plus de 7 jours et se compliquant brusquement, chez un patient présentant une forte suspicion d'infection par le VIH, l'option la plus claire est la pneumonie à P. jirovecii."}, "4": {"exist": true, "char_ranges": [[1466, 1680]], "word_ranges": [[223, 257]], "text": "un tableau respiratoire évoluant depuis plus de 7 jours et se compliquant brusquement, chez un patient présentant une forte suspicion d'infection par le VIH, l'option la plus claire est la pneumonie à P. jirovecii."}, "5": {"exist": false, "char_ranges": [], "word_ranges": [], "text": ""}}}
{"id": 76, "year": 2012, "question_id_specific": 87, "full_question": "Femme de 76 ans souffrant d'obésité et de diabète de type 2 sous traitement à la metformine. Elle s'est présentée au service des urgences depuis 3 jours avec une fièvre de 38ºC, une polyurie, une dysurie, une soif intense et une diminution progressive de l'état de conscience. L'examen physique a révélé des signes de déshydratation, une tension artérielle de 95/54 mm Hg et aucun signe de localisation neurologique. Les examens de laboratoire révèlent une leucocytose, une créatinine de 1,8 mg/dl et une glycémie de 855 mg/dl. Quel est le diagnostic le plus probable ?", "full_answer": "La question porte sur le cas d'un patient diabétique de type 2 présentant une hyperglycémie et une diminution du niveau de conscience sans focalité neurologique dans le contexte d'une infection des voies urinaires et qui a développé une insuffisance rénale aiguë d'origine pré-rénale probable. L'option 2 est immédiatement écartée car le tableau clinique n'est pas celui d'une insuffisance cardiaque. L'option 3 est également écartée parce qu'elle ne correspond pas au tableau clinique et qu'en outre, l'HSD donnerait des symptômes neurologiques avec la localisation. L'option 4 est exclue parce que l'insuffisance rénale obstructive n'entraîne pas de polyurie, de dysurie ou le reste du tableau clinique. L'option 5 est moins probable car l'acidocétose diabétique est plus typique du diabète de type 1. La bonne option est donc 1.", "type": "ANESTHÉSIOLOGIE, SOINS INTENSIFS ET MÉDECINE D'URGENCE", "options": {"1": "Coma hyperosmolaire hyperglycémique non cétosique.", "2": "Insuffisance cardiaque.", "3": "Hématome sous-dural.", "4": "Insuffisance rénale obstructive.", "5": "Acidocétose diabétique avec coma."}, "correct_option": 1, "explanations": {"1": {"exist": false, "char_ranges": [], "word_ranges": [], "text": ""}, "2": {"exist": true, "char_ranges": [[294, 400]], "word_ranges": [[44, 59]], "text": "L'option 2 est immédiatement écartée car le tableau clinique n'est pas celui d'une insuffisance cardiaque."}, "3": {"exist": true, "char_ranges": [[401, 567]], "word_ranges": [[59, 83]], "text": "L'option 3 est également écartée parce qu'elle ne correspond pas au tableau clinique et qu'en outre, l'HSD donnerait des symptômes neurologiques avec la localisation."}, "4": {"exist": true, "char_ranges": [[568, 705]], "word_ranges": [[83, 104]], "text": "L'option 4 est exclue parce que l'insuffisance rénale obstructive n'entraîne pas de polyurie, de dysurie ou le reste du tableau clinique."}, "5": {"exist": true, "char_ranges": [[706, 803]], "word_ranges": [[104, 120]], "text": "L'option 5 est moins probable car l'acidocétose diabétique est plus typique du diabète de type 1."}}}
{"id": 159, "year": 2012, "question_id_specific": 111, "full_question": "Un homme de 45 ans, ayant des antécédents de lithiase urique et de coliques néphrétiques expulsives répétées depuis 25 ans, s'est présenté aux urgences en se plaignant de palpitations et d'une douleur lombaire droite intense depuis 2 heures. L'analyse d'urine a révélé un pH de 5,5 et une leucocyturie sans protéinurie. L'ECG confirme une fibrillation auriculaire inconnue jusqu'alors. La biochimie sanguine montre une créatinine de 0,9 mg/dl, un calcium de 11 mg/dl et une LDH de 950 U/l. Quelle est la manœuvre diagnostique la plus utile à effectuer ?", "full_answer": "Pour cette question, j'ai plus de doutes quant à la réponse correcte, car nous devons interpréter quel est le test le plus utile, avec lequel nous pouvons obtenir les meilleures performances. Toutefois, cette situation dépend également des services de radiologie de chaque centre et de leurs préférences. Je me concentrerai sur la partie urologique, en laissant de côté le reste des symptômes. - La radiographie abdominale est le premier examen d'imagerie à réaliser, car elle est rapide, simple et pratiquement inoffensive (sauf chez les enfants et les femmes enceintes). Quatre-vingt-dix pour cent des calculs sont radio-opaques et donc visibles, mais la sensibilité de cet examen, utilisé isolément, tombe à 45-59 %. Il s'agit d'un examen obligatoire pour toute suspicion de douleur colique. Cependant, son rendement est inférieur à celui des autres tests. Si l'anamnèse urologique évoque une lithiase urique (radiotransparente), avec un pH acide qui confirme le diagnostic d'origine urique, on ne peut jamais exclure une lithiase d'oxalate de calcium, qui peut apparaître de façon synchrone. - L'IVUS me semble être l'une des options les plus performantes. Elle nous donnera la clarté diagnostique de l'obstruction, de son ampleur et de ses répercussions rénales. Elle est depuis longtemps considérée comme l'examen de choix pour l'étude des patients atteints de lithiase rénale. C'est un examen dont la sensibilité et la spécificité sont élevées (87-90% et 94-100%, respectivement), qui est relativement inoffensif et qui est disponible dans tous les hôpitaux. Il fournit des informations sur le nombre, la taille, la forme et la localisation du calcul, ainsi que sur l'état fonctionnel du rein affecté. - La tomodensitométrie avec renforcement des contrastes me semble être l'option la plus performante. Il permet de découvrir des lithiases que les autres examens ne trouvent pas et nous donne une image globale du reste des structures. Elle est très performante, mais aussi plus chère, plus longue et plus irradiante. Il a été confirmé qu'il s'agit d'un examen à haute sensibilité et spécificité, surpassant les tests précédents (94-100% et 92-100%, respectivement, pour la détection des calculs urétéraux) et, grâce à ses nombreux avantages, il est devenu le test de référence et devrait remplacer l'IVUS et l'échographie dans le monde entier. Il effectue un balayage hélicoïdal avec des coupes de 5 mm, ce qui permet de détecter des calculs jusqu'à 2 mm. Avec l'administration d'un produit de contraste, elle permet d'évaluer la fonction rénale. - L'échographie est un examen rapide, bon marché et sûr, mais très dépendant du radiologue. Elle donne d'excellentes informations rénales sur les répercussions obstructives, mais moins de qualité en ce qui concerne la recherche d'une lithiase. L'échographie ne détecte que les calculs de plus de 4 mm, situés dans la jonction pyélo-urétérale ou la jonction urétéro-vésicale, alors que l'uretère lombaire et pelvien est difficilement accessible en raison de l'interposition des boucles intestinales. Par conséquent, l'échographie seule n'est pas très sensible (20-45%) pour la détection des calculs, mais en combinaison avec la radiographie abdominale simple, elle augmente sa sensibilité et sa spécificité. - L'échocardiographie Doppler n'a aucune utilité dans le diagnostic urologique. Ma réponse serait la tomodensitométrie avec contraste ev et phase excrétoire (réponse (c)), en termes de performance, de sensibilité et de spécificité. Il existe des services de radiologie plus compétents ou prédisposés à l'un ou l'autre. Sans oublier qu'une radiographie abdominale doit toujours être réalisée en cas de suspicion de colique.", "type": "UROLOGIE", "options": {"1": "Radiographie abdominale simple.", "2": "IVU.", "3": "CT avec contraste.", "4": "Échographie abdominale.", "5": "Échocardiographie Doppler."}, "correct_option": 3, "explanations": {"1": {"exist": false, "char_ranges": [], "word_ranges": [], "text": ""}, "2": {"exist": true, "char_ranges": [[1098, 1383]], "word_ranges": [[167, 210]], "text": "L'IVUS me semble être l'une des options les plus performantes. Elle nous donnera la clarté diagnostique de l'obstruction, de son ampleur et de ses répercussions rénales. Elle est depuis longtemps considérée comme l'examen de choix pour l'étude des patients atteints de lithiase rénale."}, "3": {"exist": true, "char_ranges": [[1711, 1942]], "word_ranges": [[262, 298]], "text": "La tomodensitométrie avec renforcement des contrastes me semble être l'option la plus performante. Il permet de découvrir des lithiases que les autres examens ne trouvent pas et nous donne une image globale du reste des structures."}, "4": {"exist": true, "char_ranges": [[2557, 2798]], "word_ranges": [[395, 430]], "text": "L'échographie est un examen rapide, bon marché et sûr, mais très dépendant du radiologue. Elle donne d'excellentes informations rénales sur les répercussions obstructives, mais moins de qualité en ce qui concerne la recherche d'une lithiase."}, "5": {"exist": true, "char_ranges": [[3264, 3341]], "word_ranges": [[496, 505]], "text": "L'échocardiographie Doppler n'a aucune utilité dans le diagnostic urologique."}}}
{"id": 52, "year": 2011, "question_id_specific": 144, "full_question": "Un patient de 36 ans, d'une autre culture, qui vit en Espagne depuis 4 ans, se présente à la clinique du NHS. Il déclare présenter des symptômes d'anxiété et d'humeur dépressive depuis 10 mois. Cet état interfère modérément avec ses activités quotidiennes. Elle n'a pas d'antécédents psychiatriques. Cette situation résulte du décès, dans un accident de voiture, d'un frère aîné dont il était très proche. Parmi les diagnostics suivants, lequel est le plus approprié ?", "full_answer": "La relative douceur des symptômes, qui n'interfèrent pas sérieusement avec ses activités quotidiennes, exclut en principe le diagnostic de dépression majeure. Il n'y a pas de preuves de phases hypomanes qui justifieraient un trouble bipolaire II. La dysthymie nécessite une durée d'au moins deux ans. La cyclothymie est un diagnostic très rare de nos jours (car, comme l'a dit Castilla del Pino dans une interview, ce qui était autrefois des cyclothymiques est maintenant appelé bipolaire, ce qui contribue à l'une des multiples épidémies psychiatriques de notre époque), qui consiste en de légères oscillations entre les pôles dépressifs et manifestes. Mais il n'existe pas de données sur ces oscillations. La bonne réponse est très probablement 3. Le trouble adaptatif apparaît dans les 3 mois suivant l'événement stressant et ne dure pas plus de 6 mois si l'événement stressant ou ses conséquences ont cessé (ce qui n'est pas le cas puisque le frère du patient imaginaire est toujours mort). Les symptômes anxio-dépressifs légers correspondent à un tel diagnostic. Encore un exemple de la manière dont on conceptualise comme une maladie ce qui n'est qu'une réalité de la vie (même si elle est douloureuse). Le patient imaginaire prend certainement un antidépresseur parce que quelque chose peut peut-être l'aider (1).", "type": "PSYCHIATRIE", "options": {"1": "Dépression majeure.", "2": "Trouble bipolaire II.", "3": "Trouble de l'adaptation.", "4": "Dysthymie.", "5": "Cyclothymie."}, "correct_option": 3, "explanations": {"1": {"exist": true, "char_ranges": [[0, 158]], "word_ranges": [[0, 21]], "text": "La relative douceur des symptômes, qui n'interfèrent pas sérieusement avec ses activités quotidiennes, exclut en principe le diagnostic de dépression majeure."}, "2": {"exist": true, "char_ranges": [[159, 246]], "word_ranges": [[21, 36]], "text": "Il n'y a pas de preuves de phases hypomanes qui justifieraient un trouble bipolaire II."}, "3": {"exist": true, "char_ranges": [[750, 1067]], "word_ranges": [[115, 166]], "text": "Le trouble adaptatif apparaît dans les 3 mois suivant l'événement stressant et ne dure pas plus de 6 mois si l'événement stressant ou ses conséquences ont cessé (ce qui n'est pas le cas puisque le frère du patient imaginaire est toujours mort). Les symptômes anxio-dépressifs légers correspondent à un tel diagnostic."}, "4": {"exist": true, "char_ranges": [[247, 300]], "word_ranges": [[36, 45]], "text": "La dysthymie nécessite une durée d'au moins deux ans."}, "5": {"exist": true, "char_ranges": [[301, 707]], "word_ranges": [[45, 108]], "text": "La cyclothymie est un diagnostic très rare de nos jours (car, comme l'a dit Castilla del Pino dans une interview, ce qui était autrefois des cyclothymiques est maintenant appelé bipolaire, ce qui contribue à l'une des multiples épidémies psychiatriques de notre époque), qui consiste en de légères oscillations entre les pôles dépressifs et manifestes. Mais il n'existe pas de données sur ces oscillations."}}}
{"id": 297, "year": 2016, "question_id_specific": 50, "full_question": "Un homme de 58 ans a signalé des lésions cutanées progressives et légèrement douloureuses sur son bras gauche depuis plusieurs semaines. Ces lésions avaient débuté par une lésion érythémateuse sur le pouce gauche. Il présentait des stries rougeâtres visibles entre les lésions. Le patient n'avait pas de fièvre ni d'autres symptômes généraux. Il avait travaillé dans son jardin mais ne se souvenait d'aucune blessure. Le diagnostic étiologique a été établi par la culture d'une biopsie cutanée. Quel est l'agent causal le plus probable de ce processus ?", "full_answer": "Le patient exerce une activité en contact avec la terre et les plantes (qui ne sont pas fournies gratuitement). Les symptômes sont chroniques, débutant distalement (pouce gauche) et affectant progressivement le bras ipsilatéral (stries rougeâtres). Il n'y a pas de symptômes systémiques. Le diagnostic a été posé sur la base d'une biopsie cutanée. Parmi les micro-organismes mentionnés, Microsporum gypseum ne présente pas habituellement de dissémination de l'infection avec un aspect lymphangitique, en général les champignons kératinophiles (dermatophytes) n'ont pas la capacité d'envahir les tissus profonds, nous avons donc exclu cette option. Les infections à Staphylococcus aureus peuvent affecter les couches superficielles et profondes des tissus, elles peuvent se propager en suivant une voie lymphangitique, mais une évolution aussi longue n'est pas habituelle, elle est généralement aiguë (quelques jours) et s'accompagne habituellement de symptômes systémiques, c'est pourquoi nous écartons cette option. Mycobacterium marinum (et non Mycobacterum marinum) est une mycobactérie non tuberculeuse caractérisée par son caractère environnemental, opportuniste et photochromogène. Elle se développe lentement, pendant 2 à 8 semaines, à une température de 30 à 37 °C. Elle affecte différentes espèces de poissons dans les eaux froides, chaudes, douces ou salées, en particulier les eaux stagnantes des aquariums et des piscines non chlorées. L'infection chez l'homme se produit par contact direct avec des poissons ou de l'eau contaminée en présence d'une perte de continuité de la peau de l'hôte. Notre patient n'a pas d'antécédents d'exposition aux poissons ou à l'eau (ils ne le disent pas), donc si l'on considère que le diagnostic est généralement basé sur un indice de suspicion élevé et sur la demande spécifique de ces antécédents, notre cas est loin de cette possibilité. Le complexe Sporothrix schenckii est l'agent causal de la sporotrichose, ce champignon dimorphique se trouve dans le sol, le complexe se compose de 5 espèces, S. schenckii sensu strictu, S. brasiliensis et S. globosa, capables de provoquer des maladies humaines et animales ; les deux autres sont S. mexicana et S. albicana qui ne sont associées à aucune maladie. Les patients sont généralement infectés par l'exposition au sol via un objet pointu (épines, tiges, griffes d'animaux, etc.). L'évolution est lente, les tissus profonds sont touchés et la propagation suit généralement la voie décrite dans la question.", "type": "LES MALADIES INFECTIEUSES", "options": {"1": "Dermatophytose due à Microsporum gypseum.", "2": "Infection cutanée à Staphylococcus aureus.", "3": "Sporotrichose.", "4": "Infection cutanée à Mycobacterium marinum.", "5": null}, "correct_option": 3, "explanations": {"1": {"exist": true, "char_ranges": [[387, 611]], "word_ranges": [[57, 85]], "text": "Microsporum gypseum ne présente pas habituellement de dissémination de l'infection avec un aspect lymphangitique, en général les champignons kératinophiles (dermatophytes) n'ont pas la capacité d'envahir les tissus profonds,"}, "2": {"exist": true, "char_ranges": [[648, 972]], "word_ranges": [[91, 134]], "text": "Les infections à Staphylococcus aureus peuvent affecter les couches superficielles et profondes des tissus, elles peuvent se propager en suivant une voie lymphangitique, mais une évolution aussi longue n'est pas habituelle, elle est généralement aiguë (quelques jours) et s'accompagne habituellement de symptômes systémiques,"}, "3": {"exist": true, "char_ranges": [[2251, 2502]], "word_ranges": [[334, 371]], "text": "Les patients sont généralement infectés par l'exposition au sol via un objet pointu (épines, tiges, griffes d'animaux, etc.). L'évolution est lente, les tissus profonds sont touchés et la propagation suit généralement la voie décrite dans la question."}, "4": {"exist": true, "char_ranges": [[1604, 1886]], "word_ranges": [[228, 275]], "text": "Notre patient n'a pas d'antécédents d'exposition aux poissons ou à l'eau (ils ne le disent pas), donc si l'on considère que le diagnostic est généralement basé sur un indice de suspicion élevé et sur la demande spécifique de ces antécédents, notre cas est loin de cette possibilité."}, "5": {"exist": false, "char_ranges": [], "word_ranges": [], "text": ""}}}
{"id": 74, "year": 2012, "question_id_specific": 63, "full_question": "Une femme de 67 ans, ayant des antécédents d'insuffisance cardiaque congestive (sous traitement par ramipril et furosémide) et de diabète de type 2 (sous traitement par insuline), se présente au service des urgences pour dyspnée. L'analyse des gaz du sang effectuée en respirant l'air ambiant montre un pH de 7,45, une PaO2 de 56, une PaCO2 de 30 mm Hg, une HCO3 de 26 mm Hg. Quelle altération des gaz du sang la patiente présente-t-elle ?", "full_answer": "J'ai trouvé cette question un peu difficile. Les résultats séparés sont les suivants : pH normal mais avec une tendance à l'alcalose, insuffisance respiratoire aiguë, hypocapnie et bicarbonate normal. La réponse la plus appropriée me semble être 1, puisque le patient ne présente pas d'alcalose métabolique (le bicarbonate serait plus élevé), et encore moins d'acidose (le bicarbonate serait bas). L'alcalose n'est pas non plus mixte, mais principalement respiratoire. La patiente est probablement en train d'hyperventiler à cause de son insuffisance respiratoire, et le rein, qui compense toujours plus tard, commence à retenir l'HCO3 pour compenser et maintenir le pH. Je pense donc que la bonne réponse est 1.", "type": "ANESTHÉSIOLOGIE, SOINS INTENSIFS ET MÉDECINE D'URGENCE", "options": {"1": "Insuffisance respiratoire hypoxémique avec alcalose respiratoire compensée.", "2": "Alcalose métabolique aiguë avec insuffisance respiratoire hypoxémique et hypocapnique.", "3": "Insuffisance respiratoire chronique normocapnique, sans perturbation de l'équilibre acido-basique.", "4": "Insuffisance respiratoire chronique hypoxémique avec acidose métabolique compensée.", "5": "Insuffisance respiratoire hypoxémique avec alcalose mixte compensée."}, "correct_option": 1, "explanations": {"1": {"exist": true, "char_ranges": [[201, 670]], "word_ranges": [[29, 99]], "text": "La réponse la plus appropriée me semble être 1, puisque le patient ne présente pas d'alcalose métabolique (le bicarbonate serait plus élevé), et encore moins d'acidose (le bicarbonate serait bas). L'alcalose n'est pas non plus mixte, mais principalement respiratoire. La patiente est probablement en train d'hyperventiler à cause de son insuffisance respiratoire, et le rein, qui compense toujours plus tard, commence à retenir l'HCO3 pour compenser et maintenir le pH."}, "2": {"exist": false, "char_ranges": [], "word_ranges": [], "text": ""}, "3": {"exist": false, "char_ranges": [], "word_ranges": [], "text": ""}, "4": {"exist": false, "char_ranges": [], "word_ranges": [], "text": ""}, "5": {"exist": false, "char_ranges": [], "word_ranges": [], "text": ""}}}
{"id": 288, "year": 2016, "question_id_specific": 227, "full_question": "Un patient de 76 ans souffrant d'hypertension et de diabète sucré se présente au service des urgences car il présente depuis 72 heures des palpitations et une diminution de sa capacité d'effort. À son arrivée, il présente une fibrillation auriculaire avec une réponse ventriculaire d'environ 120 bpm. Lequel des éléments suivants est FAUX ?", "full_answer": "En tant que premier épisode de fibrillation auriculaire, c'est le candidat idéal pour l'ablation par cathéter.", "type": "CARDIOLOGIE ET CHIRURGIE VASCULAIRE", "options": {"1": "Ce patient doit être anticoagulé par voie orale à vie, sauf contre-indication.", "2": "Si nous décidons d'effectuer une cardioversion à l'arrivée aux urgences, une échocardiographie transœsophagienne doit être réalisée au préalable.", "3": "En tant que premier épisode de fibrillation auriculaire, c'est le candidat idéal pour l'ablation par cathéter.", "4": "Des bêta-bloquants peuvent être utilisés pour ralentir le rythme cardiaque.", "5": null}, "correct_option": 3, "explanations": {"1": {"exist": false, "char_ranges": [], "word_ranges": [], "text": ""}, "2": {"exist": false, "char_ranges": [], "word_ranges": [], "text": ""}, "3": {"exist": true, "char_ranges": [[0, 110]], "word_ranges": [[0, 16]], "text": "En tant que premier épisode de fibrillation auriculaire, c'est le candidat idéal pour l'ablation par cathéter."}, "4": {"exist": false, "char_ranges": [], "word_ranges": [], "text": ""}, "5": {"exist": false, "char_ranges": [], "word_ranges": [], "text": ""}}}
{"id": 460, "year": 2018, "question_id_specific": 174, "full_question": "Une femme de 60 ans, diabétique insulinodépendante, a présenté une omalgie nocturne prédominante du côté droit depuis plusieurs semaines. Elle ne signale aucun traumatisme. L'examen physique a révélé une limitation active et passive de toute l'amplitude des mouvements de l'épaule. Quel tableau clinique devez-vous considérer comme la première possibilité de diagnostic ?", "full_answer": "Tout fait penser à une capsulite adhésive, comme vous le dites. L'âge, le sexe, la clinique... le DM...", "type": "TRAUMATOLOGIE ET ORTHOPÉDIE", "options": {"1": "Tumeur maligne située dans l'épiphyse proximale de l'humérus.", "2": "Arthrite septique de l'épaule.", "3": "Capsulite adhésive.", "4": "Processus dégénératif acromio-claviculaire arthrosique.", "5": null}, "correct_option": 3, "explanations": {"1": {"exist": false, "char_ranges": [], "word_ranges": [], "text": ""}, "2": {"exist": false, "char_ranges": [], "word_ranges": [], "text": ""}, "3": {"exist": true, "char_ranges": [[0, 103]], "word_ranges": [[0, 18]], "text": "Tout fait penser à une capsulite adhésive, comme vous le dites. L'âge, le sexe, la clinique... le DM..."}, "4": {"exist": false, "char_ranges": [], "word_ranges": [], "text": ""}, "5": {"exist": false, "char_ranges": [], "word_ranges": [], "text": ""}}}
{"id": 333, "year": 2016, "question_id_specific": 90, "full_question": "Une femme de 35 ans a consulté pour un nodule thyroïdien droit détecté fortuitement un matin lorsqu'elle a observé une grosseur dans le miroir sur la face antérieure du cou. Après les examens nécessaires, la patiente a subi une thyroïdectomie totale avec ablation du ganglion central. Le rapport du pathologiste indique que le nodule thyroïdien de 2,3 cm était entièrement occupé par un carcinome papillaire de la thyroïde, variante à cellules hautes, sans infiltration vasculaire mais avec une infiltration capsulaire. Les concentrations de thyroglobuline 24 heures après la thyroïdectomie sont de 14 ng/mL. Quelle serait la prochaine étape pour ce patient ?", "full_answer": "La recherche de la thyroglobuline dans le sang et la mise au point de techniques permettant de la déterminer ont fourni une nouvelle procédure de suivi des patients atteints de carcinome papillaire ou folliculaire de la thyroïde. La thyroglobuline est synthétisée dans les cellules folliculaires puis sécrétée dans la circulation. Après une thyroïdectomie totale, elle devrait donc disparaître ; si ce n'est pas le cas, il faut envisager l'existence de restes de tissu métastatique.", "type": "ENDOCRINOLOGIE", "options": {"1": "Renvoyez le patient au chirurgien pour qu'il effectue un drainage du ganglion lymphatique latérocervical droit.", "2": "Commencer le traitement avec une dose de lévothyroxine supprimant la TSH et prévoir un examen 6 mois plus tard avec un nouveau dosage analytique de la thyroglobuline et une échographie du col de l'utérus.", "3": "Demander un PET-CT scan pour vérifier l'atteinte des ganglions lymphatiques.", "4": "Retarder le début du traitement de substitution de la TSH par la lévothyroxine et adresser le patient au service de médecine nucléaire pour l'administration d'une dose ablative de 100 mCi d'I131.", "5": null}, "correct_option": 4, "explanations": {"1": {"exist": false, "char_ranges": [], "word_ranges": [], "text": ""}, "2": {"exist": false, "char_ranges": [], "word_ranges": [], "text": ""}, "3": {"exist": false, "char_ranges": [], "word_ranges": [], "text": ""}, "4": {"exist": true, "char_ranges": [[229, 482]], "word_ranges": [[36, 74]], "text": "thyroïde. La thyroglobuline est synthétisée dans les cellules folliculaires puis sécrétée dans la circulation. Après une thyroïdectomie totale, elle devrait donc disparaître ; si ce n'est pas le cas, il faut envisager l'existence de restes de tissu métastatique."}, "5": {"exist": false, "char_ranges": [], "word_ranges": [], "text": ""}}}
{"id": 185, "year": 2013, "question_id_specific": 66, "full_question": "Un patient de 74 ans, ayant subi une résection intestinale à la suite d'une thrombose mésentérique, vient en consultation pour fatigue et douleurs osseuses. Il avait subi une fracture de Colles trois mois auparavant. Les examens de laboratoire montrent une anémie de 9,5 g/dl, microcytaire, une albumine de 3,5 g/di, un calcium de 7,5 mg/dl, un phosphate de 2,0 mg/dl et une phosphatase alcaline de 224 UI (normale jusqu'à 120 UI). La radiographie du fémur montre des bandes radiotransparentes perpendiculaires à la corticale sur la face interne de la partie supérieure du fémur. Quel est le diagnostic le plus probable ?", "full_answer": "Bien que celui-ci ne m'ait pas été attribué, j'y réponds car il traite d'un sujet d'endocrinologie : la vitamine D. Dans ce cas, il y a des données importantes comme l'histoire de la résection intestinale, l'hypocalcémie et l'hypophosphorémie ; cela devrait nous amener à penser à un déficit en vitamine D dû à la malabsorption et donc à l'ostéomalacie. Le reste des données : l'antécédent de fracture, la douleur et la radiographie confirment le diagnostic.", "type": "ENDOCRINOLOGIE", "options": {"1": "Ostéoporose.", "2": "Maladie de Paget.", "3": "Ostéite fibrosante.", "4": "Métastases du cancer de la prostate.", "5": "Ostéomalacie."}, "correct_option": 5, "explanations": {"1": {"exist": false, "char_ranges": [], "word_ranges": [], "text": ""}, "2": {"exist": false, "char_ranges": [], "word_ranges": [], "text": ""}, "3": {"exist": false, "char_ranges": [], "word_ranges": [], "text": ""}, "4": {"exist": false, "char_ranges": [], "word_ranges": [], "text": ""}, "5": {"exist": true, "char_ranges": [[115, 353]], "word_ranges": [[19, 59]], "text": "D. Dans ce cas, il y a des données importantes comme l'histoire de la résection intestinale, l'hypocalcémie et l'hypophosphorémie ; cela devrait nous amener à penser à un déficit en vitamine D dû à la malabsorption et donc à l'ostéomalacie."}}}
{"id": 107, "year": 2012, "question_id_specific": 206, "full_question": "Une fillette de 8 ans (cas index) reçoit un diagnostic clinique de neurofibromatose de type 1 (NF1) ou maladie de Von Recklinghausen avec de multiples neurofibromes, des taches café-au-lait et des nodules de Lisch. Son père (non diagnostiqué NF1) est décédé dans un accident de la route à l'âge de 38 ans. À l'examen, la mère présente deux taches café-au-lait et assiste à la consultation de conseil génétique avec son nouveau partenaire où un diagnostic génétique préimplantatoire (DPI) est envisagé. Le DPI est-il indiqué dans ce cas ?", "full_answer": "La première étape consiste à détecter la mutation et, une fois celle-ci détectée, si la mère est porteuse, à procéder à un DPI.", "type": "GÉNÉTIQUE ET IMMUNOLOGIE", "options": {"1": "Oui, comme la mère a deux taches café-au-lait, elle est porteuse et le DPI est indiqué avec ces données.", "2": "Elle est indiquée après la détection de la mutation causale chez le cas index et éventuellement chez sa mère.", "3": "Non indiqué car la NF1 répond à des mutations dans le gène de la neurofibromine (17q11.2) qui se transmet de façon récessive.", "4": "Non, deux taches café-au-lait ne sont pas diagnostiques et il est très peu probable que votre nouveau partenaire soit porteur (la NF1 est une maladie rare).", "5": "Au vu de ces données, un DPI est indiqué, consistant à sélectionner des embryons in vitro, afin d'implanter dans l'utérus maternel ceux qui ne présentent pas la mutation."}, "correct_option": 2, "explanations": {"1": {"exist": false, "char_ranges": [], "word_ranges": [], "text": ""}, "2": {"exist": true, "char_ranges": [[0, 127]], "word_ranges": [[0, 23]], "text": "La première étape consiste à détecter la mutation et, une fois celle-ci détectée, si la mère est porteuse, à procéder à un DPI."}, "3": {"exist": false, "char_ranges": [], "word_ranges": [], "text": ""}, "4": {"exist": false, "char_ranges": [], "word_ranges": [], "text": ""}, "5": {"exist": false, "char_ranges": [], "word_ranges": [], "text": ""}}}
{"id": 114, "year": 2012, "question_id_specific": 96, "full_question": "Une femme de 43 ans a consulté son médecin traitant pour fatigue, gingivorrhée et pétéchies. Les tests de laboratoire ont montré une anémie de 8g/dl, une thrombocytopénie de 4000/microlitre et une leucopénie de 1200/microlitre avec une neutropénie absolue. Une étude de la coagulation a montré un allongement du TCA (43″), une diminution de l'activité prothrombique (55%), une hypofibrinogénémie (98 mg/dl) et la présence de concentrations élevées de D-Dimer et de monomères de fibrine. Le patient a été référé pour un examen hématologique urgent et une aspiration de la moelle osseuse a montré une infiltration massive par des éléments immatures avec des noyaux clivés et des bâtonnets d'Auer dans le cytoplasme. Quel est le diagnostic le plus probable chez ce patient ?", "full_answer": "Après avoir lu la longue déclaration, s'être essoufflé, l'avoir parcourue et s'être laissé submerger, il est temps de lui donner un visage et de s'emparer de quelque chose de très caractéristique. Une femme atteinte de pancytopénie. Une moelle osseuse qui présente une infiltration massive par des éléments immatures qui prolifèrent sans arrêt... avec des cannes d'Auer ? Leucémie promyélocytaire aiguë. Ces bâtonnets sont la fusion des granules primaires des myélocytes en développement. Robin & Cotran le disent à la page 696 de leur livre \"General Pathology\" [1]. C'est également ce que disent Carr et Rodak dans leur livre \"Atlas of clinical haematology\" à la page 157 [2].", "type": "HÉMATOLOGIE", "options": {"1": "Leucémie myéloïde chronique.", "2": "Leucémie promyélocytaire aiguë.", "3": "Leucémie monoblastique aiguë.", "4": "Leucémie lymphoïde chronique.", "5": "Leucémie lymphoïde aiguë."}, "correct_option": 2, "explanations": {"1": {"exist": false, "char_ranges": [], "word_ranges": [], "text": ""}, "2": {"exist": true, "char_ranges": [[197, 488]], "word_ranges": [[31, 72]], "text": "Une femme atteinte de pancytopénie. Une moelle osseuse qui présente une infiltration massive par des éléments immatures qui prolifèrent sans arrêt... avec des cannes d'Auer ? Leucémie promyélocytaire aiguë. Ces bâtonnets sont la fusion des granules primaires des myélocytes en développement."}, "3": {"exist": false, "char_ranges": [], "word_ranges": [], "text": ""}, "4": {"exist": false, "char_ranges": [], "word_ranges": [], "text": ""}, "5": {"exist": false, "char_ranges": [], "word_ranges": [], "text": ""}}}
{"id": 61, "year": 2011, "question_id_specific": 92, "full_question": "Un homme de 30 ans est tombé d'une hauteur de 2 mètres sur ses pieds. À son arrivée au service des urgences, il était couché sur le dos et sa vessie avait été sondée avec des urines claires. Il présente un gonflement du talon gauche, ainsi que de légers picotements de la face antérieure de la cuisse droite et du scrotum, et une dysesthésie des deux pieds. Le patient dit au revoir au brancardier en lui serrant la main et en lui tapotant simultanément l'épaule. La rotation du MMII sur le brancard saisi par les chevilles est indolore et la compression du bassin est asymptomatique. Suivant :", "full_answer": "D'après les symptômes rapportés, fourmillements dans la partie antérieure de la cuisse, dans le scrotum et dysesthésie dans les pieds, il semble y avoir une atteinte vertébrale au niveau de L2-3 et des racines sacrées, de sorte que la bonne chose à faire serait de procéder à un examen radiologique sans mobiliser le patient. Cependant, le patient devrait être mobilisé pour un examen en bloc. La colonne cervicale semble indemne car le patient mobilise correctement ses deux bras, sans paresthésie.", "type": "ANESTHÉSIOLOGIE ET SOINS INTENSIFS", "options": {"1": "Vous serez assis sur le brancard afin que la colonne cervicale puisse être explorée.", "2": "Se référer à la prescription pour l'étude des extrémités inférieures.", "3": "Palper l'abdomen et le tourner en bloc pour palper la colonne thoraco-lombaire.", "4": "Vous serez placé dans un collier cervical et envoyé en radiologie pour une imagerie de la colonne cervicale et des extrémités inférieures.", "5": "Nous demanderons une échographie abdominale."}, "correct_option": 3, "explanations": {"1": {"exist": false, "char_ranges": [], "word_ranges": [], "text": ""}, "2": {"exist": false, "char_ranges": [], "word_ranges": [], "text": ""}, "3": {"exist": true, "char_ranges": [[337, 499]], "word_ranges": [[55, 80]], "text": "le patient devrait être mobilisé pour un examen en bloc. La colonne cervicale semble indemne car le patient mobilise correctement ses deux bras, sans paresthésie."}, "4": {"exist": false, "char_ranges": [], "word_ranges": [], "text": ""}, "5": {"exist": false, "char_ranges": [], "word_ranges": [], "text": ""}}}
{"id": 231, "year": 2014, "question_id_specific": 180, "full_question": "Un nourrisson âgé d'un mois s'est présenté au service des urgences de l'hôpital, adressé par son pédiatre pour une jaunisse. Il souffrait d'une jaunisse depuis 10 jours, qui allait en augmentant. Les analyses sanguines ont révélé un taux de bilirubine totale de 7 mg/dl et un taux de bilirubine indirecte de 1,5 mg/dl. La cause la plus probable de cet ictère est l'une des suivantes :", "full_answer": "L'augmentation de la bilirubine directe exclut immédiatement toutes les autres options où l'augmentation de la bilirubine totale se ferait au détriment de la bilirubine indirecte. L'atrésie des voies biliaires est la principale indication de la transplantation hépatique chez le nourrisson.", "type": "PÉDIATRIE", "options": {"1": "Atrésie des voies biliaires.", "2": "Ictère d'allaitement.", "3": "Iso-immunisation 0-A tardive.", "4": "La maladie de Gilbert.", "5": "Sphérocytose héréditaire."}, "correct_option": 1, "explanations": {"1": {"exist": true, "char_ranges": [[180, 290]], "word_ranges": [[25, 40]], "text": "L'atrésie des voies biliaires est la principale indication de la transplantation hépatique chez le nourrisson."}, "2": {"exist": true, "char_ranges": [[0, 179]], "word_ranges": [[0, 25]], "text": "L'augmentation de la bilirubine directe exclut immédiatement toutes les autres options où l'augmentation de la bilirubine totale se ferait au détriment de la bilirubine indirecte."}, "3": {"exist": true, "char_ranges": [[0, 179]], "word_ranges": [[0, 25]], "text": "L'augmentation de la bilirubine directe exclut immédiatement toutes les autres options où l'augmentation de la bilirubine totale se ferait au détriment de la bilirubine indirecte."}, "4": {"exist": true, "char_ranges": [[0, 179]], "word_ranges": [[0, 25]], "text": "L'augmentation de la bilirubine directe exclut immédiatement toutes les autres options où l'augmentation de la bilirubine totale se ferait au détriment de la bilirubine indirecte."}, "5": {"exist": true, "char_ranges": [[0, 179]], "word_ranges": [[0, 25]], "text": "L'augmentation de la bilirubine directe exclut immédiatement toutes les autres options où l'augmentation de la bilirubine totale se ferait au détriment de la bilirubine indirecte."}}}
{"id": 87, "year": 2012, "question_id_specific": 54, "full_question": "Un patient de 81 ans consulte pour un épisode de syncope. Il se plaint de dyspnée à l'effort depuis un an. L'auscultation cardiaque révèle un souffle systolique 3/6 sur le bord sternal gauche, irradiant vers les carotides et la pointe. Quelle pathologie vous semble la plus probable ?", "full_answer": "Question très simple. Vous décrivez le \"portrait\" d'une sténose aortique dégénérative... Un patient âgé présentant des symptômes de dyspnée et de syncope, ainsi qu'un souffle systolique irradiant vers les carotides et la pointe, autant d'éléments typiques de cette maladie.", "type": "CARDIOLOGIE ET CHIRURGIE VASCULAIRE", "options": {"1": "Bloc auriculo-ventriculaire du troisième degré.", "2": "Sténose aortique dégénérative.", "3": "Insuffisance de la valve mitrale.", "4": "Cardiomyopathie hypertrophique.", "5": "Cardiomyopathie dilatée."}, "correct_option": 2, "explanations": {"1": {"exist": false, "char_ranges": [], "word_ranges": [], "text": ""}, "2": {"exist": true, "char_ranges": [[89, 273]], "word_ranges": [[11, 39]], "text": "Un patient âgé présentant des symptômes de dyspnée et de syncope, ainsi qu'un souffle systolique irradiant vers les carotides et la pointe, autant d'éléments typiques de cette maladie."}, "3": {"exist": false, "char_ranges": [], "word_ranges": [], "text": ""}, "4": {"exist": false, "char_ranges": [], "word_ranges": [], "text": ""}, "5": {"exist": false, "char_ranges": [], "word_ranges": [], "text": ""}}}
{"id": 102, "year": 2012, "question_id_specific": 87, "full_question": "Femme de 76 ans souffrant d'obésité et de diabète de type 2 sous traitement à la metformine. Elle s'est présentée au service des urgences depuis 3 jours avec une fièvre de 38ºC, une pollakiurie, une dysurie, une soif intense et une diminution progressive de son niveau de conscience. L'examen physique montre des signes de déshydratation, une tension artérielle de 95/54 mmHg et aucun signe de focalité neurologique. Les examens de laboratoire révèlent une leucocytose et une créatinine de 1,8 mg/dL. Quel est le diagnostic le plus probable ?", "full_answer": "Question facile sur les complications aiguës du diabète sucré.", "type": "ENDOCRINOLOGIE", "options": {"1": "Coma hyperosmolaire hyperglycémique non cétosique.", "2": "Insuffisance cardiaque.", "3": "Hématome sous-dural.", "4": "Insuffisance rénale obstructive.", "5": "Acidocétose diabétique avec coma."}, "correct_option": 1, "explanations": {"1": {"exist": false, "char_ranges": [], "word_ranges": [], "text": ""}, "2": {"exist": false, "char_ranges": [], "word_ranges": [], "text": ""}, "3": {"exist": false, "char_ranges": [], "word_ranges": [], "text": ""}, "4": {"exist": false, "char_ranges": [], "word_ranges": [], "text": ""}, "5": {"exist": false, "char_ranges": [], "word_ranges": [], "text": ""}}}
{"id": 512, "year": 2021, "question_id_specific": 153, "full_question": "Le patient a été diagnostiqué avec une diverticulite aiguë, traitée avec de l'amoxicilline/acide clavulanique. Après 5 jours, il a commencé à avoir de la fièvre et de la diarrhée. On lui a diagnostiqué une colite à Clostridium difficile, l'amoxicilline/acide clavulanique a été arrêtée et du métronidazole lui a été prescrit. Après 4 jours, il n'a pas répondu au métronidazole, mais son état clinique est stable. L'étape suivante du traitement est la suivante :", "full_answer": "Le traitement de choix de l'infection à Clostridioides difficile est la vancomycine, administrée par voie orale à raison de 125 mg/6 h pendant 10 jours. Le métronidazole a été relégué aux situations où la vancomycine n'est pas disponible.", "type": "DIGESTIF", "options": {"1": "Passer du métronidazole à la vancomycine orale (125 mg, quatre fois par jour pendant 10 jours).", "2": "Passer du métronidazole à la pipéracilline/tazobactam (4 g de pipéracilline/0,5 g de tazobactam toutes les 8 heures).", "3": "Indiquer une colectomie subtotale.", "4": "Réaliser une iléostomie à anse avec lavage antérograde au polyéthylène glycol.", "5": null}, "correct_option": 1, "explanations": {"1": {"exist": true, "char_ranges": [[0, 152]], "word_ranges": [[0, 25]], "text": "Le traitement de choix de l'infection à Clostridioides difficile est la vancomycine, administrée par voie orale à raison de 125 mg/6 h pendant 10 jours."}, "2": {"exist": false, "char_ranges": [], "word_ranges": [], "text": ""}, "3": {"exist": false, "char_ranges": [], "word_ranges": [], "text": ""}, "4": {"exist": false, "char_ranges": [], "word_ranges": [], "text": ""}, "5": {"exist": false, "char_ranges": [], "word_ranges": [], "text": ""}}}
{"id": 218, "year": 2014, "question_id_specific": 123, "full_question": "Un homme de 30 ans sans antécédents particuliers. Il est venu consulter en raison de la présence de petites lésions érythémato-violacées qui semblaient se soulever à la palpation dans la région prétibiale. Les analyses de sang et de coagulation n'ont révélé aucune anomalie, et les analyses biochimiques, de créatinine et d'ions étaient également dans les limites de la normale. L'étude du sédiment urinaire a montré une hématurie, pour laquelle le patient avait déjà été étudié à d'autres occasions, sans obtenir de diagnostic définitif. En ce qui concerne l'entité que vous suspectez dans ce cas, il est FAUX que :", "full_answer": "Il s'agit très probablement d'une glomérulonéphrite mésangiale à IgA ou d'une maladie de Berger. Nous allons donc écarter les options une par une : 1 : Vrai. L'élévation du taux d'IgA sérique est observée dans 50 % des cas. 2 : Vrai. Des dépôts mésangiaux d'IgA sont présents dans presque 100 % des cas. 3 : Cette option est fausse, car cette glomérulonéphrite se manifeste classiquement par un syndrome néphritique et non par un syndrome néphrotique (bien que dans quelques rares cas, une protéinurie de l'ordre du syndrome néphrotique apparaisse, mais le MIR ne pose pas de question sur ces rares cas). 4 : Au début, cette option a suscité des doutes dans mon esprit, mais en regardant la littérature, il est vrai que l'évolution vers l'insuffisance rénale (selon les dernières séries) se produit dans environ 25 % des cas, donc cette option est vraie. 5 : La biopsie cutanée, parce qu'elle est plus facile à réaliser que la biopsie rénale, est la technique diagnostique de choix (les lésions cutanées qui constituent le purpura de Schonlein-Henoch, si souvent associé à cette entité et que présente le patient dans le cas présent, sont biopsiées).", "type": "NEPHROLOGIE", "options": {"1": "Dans 20 à 50 % des cas, on observe une concentration élevée d'IgA dans le sérum.", "2": "La biopsie rénale révèle la présence de dépôts mésangiaux d'IgA.", "3": "Une protéinurie de l'ordre de la néphrologie est fréquente.", "4": "Elle est considérée comme une entité bénigne car moins d'un tiers des patients évoluent vers l'insuffisance rénale.", "5": "Une biopsie de la peau permet d'établir le diagnostic dans près de la moitié des cas."}, "correct_option": 3, "explanations": {"1": {"exist": true, "char_ranges": [[148, 223]], "word_ranges": [[24, 39]], "text": "1 : Vrai. L'élévation du taux d'IgA sérique est observée dans 50 % des cas."}, "2": {"exist": true, "char_ranges": [[224, 303]], "word_ranges": [[39, 54]], "text": "2 : Vrai. Des dépôts mésangiaux d'IgA sont présents dans presque 100 % des cas."}, "3": {"exist": true, "char_ranges": [[304, 450]], "word_ranges": [[54, 76]], "text": "3 : Cette option est fausse, car cette glomérulonéphrite se manifeste classiquement par un syndrome néphritique et non par un syndrome néphrotique"}, "4": {"exist": true, "char_ranges": [[605, 854]], "word_ranges": [[102, 145]], "text": "4 : Au début, cette option a suscité des doutes dans mon esprit, mais en regardant la littérature, il est vrai que l'évolution vers l'insuffisance rénale (selon les dernières séries) se produit dans environ 25 % des cas, donc cette option est vraie."}, "5": {"exist": true, "char_ranges": [[855, 1150]], "word_ranges": [[145, 193]], "text": "5 : La biopsie cutanée, parce qu'elle est plus facile à réaliser que la biopsie rénale, est la technique diagnostique de choix (les lésions cutanées qui constituent le purpura de Schonlein-Henoch, si souvent associé à cette entité et que présente le patient dans le cas présent, sont biopsiées)."}}}
{"id": 493, "year": 2020, "question_id_specific": 107, "full_question": "Un patient de 18 ans s'est présenté aux urgences pour une épistaxis qui durait depuis plusieurs jours, sans antécédents personnels ou familiaux intéressants. L'examen a révélé un état afébrile, de multiples ecchymoses et l'absence de splénomégalie palpable. Examens de laboratoire : leucocytes 7,2 x103/μL, Hb 12,3 g/dL, plaquettes 6,0 x103/μL. La thrombocytopénie a été confirmée au frottis, où l'on a observé une augmentation de la taille des plaquettes. Les études de coagulation et de biochimie sont normales. Quel est le diagnostic le plus probable ?", "full_answer": "S'ils voulaient que nous répondions qu'il s'agit d'un PTT (réponse 1), ils associeraient probablement une anémie et nous parleraient probablement de schistocytes dans le frottis. Sans compter que les symptômes du patient iraient probablement bien au-delà de l'épistaxis. Pour les deux autres réponses, il n'y a pas lieu d'hésiter. Rien ne l'indique.", "type": "HÉMATOLOGIE", "options": {"1": "Purpura thrombocytopénique thrombotique.", "2": "Coagulation intravasculaire disséminée.", "3": "Thrombocytopénie d'origine infectieuse.", "4": "Thrombocytopénie immunitaire primaire.", "5": null}, "correct_option": 4, "explanations": {"1": {"exist": true, "char_ranges": [[0, 178]], "word_ranges": [[0, 25]], "text": "S'ils voulaient que nous répondions qu'il s'agit d'un PTT (réponse 1), ils associeraient probablement une anémie et nous parleraient probablement de schistocytes dans le frottis."}, "2": {"exist": true, "char_ranges": [[271, 349]], "word_ranges": [[38, 52]], "text": "Pour les deux autres réponses, il n'y a pas lieu d'hésiter. Rien ne l'indique."}, "3": {"exist": true, "char_ranges": [[271, 349]], "word_ranges": [[38, 52]], "text": "Pour les deux autres réponses, il n'y a pas lieu d'hésiter. Rien ne l'indique."}, "4": {"exist": false, "char_ranges": [], "word_ranges": [], "text": ""}, "5": {"exist": false, "char_ranges": [], "word_ranges": [], "text": ""}}}
{"id": 566, "year": 2022, "question_id_specific": 149, "full_question": "Une femme de 67 ans souffre d'une maladie rénale chronique secondaire à une néphropathie diabétique. Sa créatinine est actuellement de 3,2 mg/dl et son débit de filtration glomérulaire estimé (CKD-EPI) est de 14 ml/min. Laquelle des affirmations suivantes est vraie ?", "full_answer": "Un taux de filtration glomérulaire inférieur à 15 ml/min est classé au stade 5 (G5), indiquant une insuffisance rénale terminale et la mise en place d'un traitement de substitution doit être envisagée (option 2 vraie, option 1 fausse). Dans cette situation, un bon contrôle de la glycémie ne devrait pas inverser les lésions rénales, bien qu'il puisse contribuer à en retarder la progression (faux choix 3). Ces patients présentent généralement une albuminurie de stade A3 (supérieure à 300 mg/24 h), et peuvent atteindre le stade néphrotique (fausse réponse 4).", "type": "NEPHROLOGIE", "options": {"1": "Il souffre d'une maladie rénale chronique de stade 3 et doit être mis sous chélateurs du phosphore.", "2": "Il souffre d'une maladie rénale chronique de stade 5 et une thérapie de remplacement rénal doit être envisagée.", "3": "Un bon contrôle de la glycémie peut inverser le processus d'insuffisance rénale.", "4": "Une protéinurie supérieure à 500 mg/24 h est peu probable.", "5": null}, "correct_option": 2, "explanations": {"1": {"exist": true, "char_ranges": [[0, 235]], "word_ranges": [[0, 38]], "text": "Un taux de filtration glomérulaire inférieur à 15 ml/min est classé au stade 5 (G5), indiquant une insuffisance rénale terminale et la mise en place d'un traitement de substitution doit être envisagée (option 2 vraie, option 1 fausse)."}, "2": {"exist": true, "char_ranges": [[0, 235]], "word_ranges": [[0, 38]], "text": "Un taux de filtration glomérulaire inférieur à 15 ml/min est classé au stade 5 (G5), indiquant une insuffisance rénale terminale et la mise en place d'un traitement de substitution doit être envisagée (option 2 vraie, option 1 fausse)."}, "3": {"exist": true, "char_ranges": [[236, 407]], "word_ranges": [[38, 66]], "text": "Dans cette situation, un bon contrôle de la glycémie ne devrait pas inverser les lésions rénales, bien qu'il puisse contribuer à en retarder la progression (faux choix 3)."}, "4": {"exist": true, "char_ranges": [[408, 562]], "word_ranges": [[66, 89]], "text": "Ces patients présentent généralement une albuminurie de stade A3 (supérieure à 300 mg/24 h), et peuvent atteindre le stade néphrotique (fausse réponse 4)."}, "5": {"exist": false, "char_ranges": [], "word_ranges": [], "text": ""}}}
{"id": 254, "year": 2014, "question_id_specific": 89, "full_question": "Un garçon de 17 ans, sportif sans antécédents, consulte son médecin pour un ictère conjonctival occasionnel. Il nie avoir pris des médicaments et avoir ressenti des douleurs abdominales. Les examens de laboratoire révèlent des taux normaux d'ALT, d'AST, de GGT et de FA, ainsi qu'une bilirubine totale de 3,2 mg/dl et une bilirubine directe de 0,4 mg/dl. Il n'y a pas d'anémie et le foie est normal à l'échographie. Quel est votre diagnostic ?", "full_answer": "Une augmentation de la bilirubine indirecte inférieure à 5 (par intermittence, en cas de stress) chez un jeune garçon en bonne santé doit faire penser au syndrome de Gilbert. Dans le syndrome de Rotor, le syndrome de Dubin-Johnson et la cholédocholithiase, on observe une augmentation de la bilirubine directe. En cas d'hépatite aiguë, il doit y avoir une élévation des transaminases.", "type": "SYSTÈME DIGESTIF", "options": {"1": "Syndrome du rotor.", "2": "Cholédocholithiase.", "3": "Syndrome de Dubin-Johnson.", "4": "Hépatite aiguë.", "5": "Syndrome de Gilbert."}, "correct_option": 5, "explanations": {"1": {"exist": true, "char_ranges": [[175, 310]], "word_ranges": [[29, 49]], "text": "Dans le syndrome de Rotor, le syndrome de Dubin-Johnson et la cholédocholithiase, on observe une augmentation de la bilirubine directe."}, "2": {"exist": true, "char_ranges": [[175, 310]], "word_ranges": [[29, 49]], "text": "Dans le syndrome de Rotor, le syndrome de Dubin-Johnson et la cholédocholithiase, on observe une augmentation de la bilirubine directe."}, "3": {"exist": true, "char_ranges": [[175, 310]], "word_ranges": [[29, 49]], "text": "Dans le syndrome de Rotor, le syndrome de Dubin-Johnson et la cholédocholithiase, on observe une augmentation de la bilirubine directe."}, "4": {"exist": true, "char_ranges": [[311, 384]], "word_ranges": [[49, 61]], "text": "En cas d'hépatite aiguë, il doit y avoir une élévation des transaminases."}, "5": {"exist": false, "char_ranges": [], "word_ranges": [], "text": ""}}}
{"id": 597, "year": 2022, "question_id_specific": 180, "full_question": "Une femme de 26 ans, chez qui on a diagnostiqué un lupus érythémateux disséminé et qui est traitée à l'hydroxychloroquine, consulte pour une sensation de faiblesse généralisée qui s'est progressivement développée au cours des 15 derniers jours. L'examen physique a révélé une pâleur cutanée et une analyse de sang a montré une Hb de 7,4 g/dL, une Hct de 31 % et une MCV de 108. Lequel des tests suivants serait le plus utile pour décider de la marche à suivre ?", "full_answer": "Dans cette question, on nous présente un patient atteint de lupus érythémateux disséminé qui présente une anémie hémolytique auto-immune. Dans ce cas, la meilleure option serait de demander des réticulocytes, mais comme cela ne figure pas parmi les options, nous devrions demander un test de Coombs, de sorte que la réponse correcte est 2. Bien que l'on veuille nous embrouiller avec un MCV 108, de sorte que nous pensions à une anémie mégaloblastique, le MCV n'est pas excessivement élevé, l'apparition de l'anémie chez notre patient a été rapide et ne correspondrait pas au LED ou à la forme de présentation. Les anticorps antinucléaires sont utilisés pour le diagnostic, mais pas pour le suivi du LED.", "type": "RHEUMATOLOGIE", "options": {"1": "Haptoglobine.", "2": "Test de Coombs.", "3": "Vitamine B12.", "4": "Anticorps antinucléaires.", "5": null}, "correct_option": 2, "explanations": {"1": {"exist": false, "char_ranges": [], "word_ranges": [], "text": ""}, "2": {"exist": true, "char_ranges": [[0, 339]], "word_ranges": [[0, 54]], "text": "Dans cette question, on nous présente un patient atteint de lupus érythémateux disséminé qui présente une anémie hémolytique auto-immune. Dans ce cas, la meilleure option serait de demander des réticulocytes, mais comme cela ne figure pas parmi les options, nous devrions demander un test de Coombs, de sorte que la réponse correcte est 2."}, "3": {"exist": true, "char_ranges": [[340, 610]], "word_ranges": [[54, 100]], "text": "Bien que l'on veuille nous embrouiller avec un MCV 108, de sorte que nous pensions à une anémie mégaloblastique, le MCV n'est pas excessivement élevé, l'apparition de l'anémie chez notre patient a été rapide et ne correspondrait pas au LED ou à la forme de présentation."}, "4": {"exist": true, "char_ranges": [[611, 704]], "word_ranges": [[100, 115]], "text": "Les anticorps antinucléaires sont utilisés pour le diagnostic, mais pas pour le suivi du LED."}, "5": {"exist": false, "char_ranges": [], "word_ranges": [], "text": ""}}}
{"id": 571, "year": 2022, "question_id_specific": 170, "full_question": "Une femme de 75 ans sous traitement à la sertraline s'est présentée au service des urgences avec des symptômes de confusion. Aucun œdème n'a été observé et la pression artérielle était de 130/70. L'analyse sanguine montre un Na de 126 mEq/l et un K de 4 mEq/l, la natriurie est de 45 mEq/l et la prise de diurétiques a été exclue. Laquelle des approches suivantes est la plus correcte ?", "full_answer": "Nous sommes en présence d'un patient présentant une hyponatrémie euvolémique modérée, suggérant un syndrome d'antidiurèse inadéquate (SADI) secondaire à la sertraline, bien que l'osmolalité urinaire soit nécessaire pour confirmer cette hypothèse. L'insuffisance surrénale primaire provoque une hyponatrémie hypovolémique (option 1 incorrecte). Le diabète insipide est l'opposé de cette pathologie, se présentant généralement avec une hypernatrémie (option 4 incorrecte). La question reste de savoir s'il faut traiter de manière agressive avec une solution saline hypertonique ou s'il faut être plus conservateur en indiquant une restriction hydrique. Avec une natrémie supérieure à 125 mmol/l, il n'est pas indiqué de traiter avec une solution saline hypertonique à moins que le patient ne présente des symptômes neurologiques graves, ce qui ne semble pas être le cas (ils décrivent un tableau confusionnel qui correspondrait à une encéphalopathie hyponatrémique modérée), et il n'est pas non plus indiqué de rétablir la natrémie dès que possible (en raison du risque de syndrome de démyélinisation osmotique). Par conséquent, la réponse correcte serait 3. Ce que je n'aime pas dans cette réponse, c'est que la deuxième partie (\"si la natrémie ne remonte pas, passer à une perfusion lente de sérum physiologique\") est incorrecte, car dans ce cas, il faut envisager l'utilisation de furosémide, de tolvaptan ou d'urée, et non de sérum physiologique (et cela n'indique pas non plus la tonicité, car isotonique n'est pas la même chose qu'hypertonique). Quoi qu'il en soit, je ne pense pas qu'il soit très probable que l'on passe outre.", "type": "NEPHROLOGIE", "options": {"1": "Il s'agit de la maladie d'Addison et des corticostéroïdes doivent être administrés immédiatement.", "2": "Administrer une solution saline hypertonique pour rétablir la natrémie dès que possible.", "3": "Indiquer une restriction hydrique et, si la natrémie n'augmente pas, passer à une perfusion lente de sérum physiologique.", "4": "Demander une IRM du cerveau, car il s'agit probablement d'un diabète insipide.", "5": null}, "correct_option": 3, "explanations": {"1": {"exist": true, "char_ranges": [[247, 343]], "word_ranges": [[31, 41]], "text": "L'insuffisance surrénale primaire provoque une hyponatrémie hypovolémique (option 1 incorrecte)."}, "2": {"exist": true, "char_ranges": [[651, 1110]], "word_ranges": [[85, 157]], "text": "Avec une natrémie supérieure à 125 mmol/l, il n'est pas indiqué de traiter avec une solution saline hypertonique à moins que le patient ne présente des symptômes neurologiques graves, ce qui ne semble pas être le cas (ils décrivent un tableau confusionnel qui correspondrait à une encéphalopathie hyponatrémique modérée), et il n'est pas non plus indiqué de rétablir la natrémie dès que possible (en raison du risque de syndrome de démyélinisation osmotique)."}, "3": {"exist": false, "char_ranges": [], "word_ranges": [], "text": ""}, "4": {"exist": true, "char_ranges": [[344, 470]], "word_ranges": [[41, 58]], "text": "Le diabète insipide est l'opposé de cette pathologie, se présentant généralement avec une hypernatrémie (option 4 incorrecte)."}, "5": {"exist": false, "char_ranges": [], "word_ranges": [], "text": ""}}}
{"id": 466, "year": 2020, "question_id_specific": 91, "full_question": "Une fille préménarchique âgée de 11 ans et 6 mois présentant une scoliose idiopathique connue. La radiographie du rachis réalisée il y a 6 mois montrait une courbure thoraco-lombaire gauche de 18° (angle de Cobb). La radiographie actuelle montre un angle de Cobb de 28°, avec une maturation de la crête iliaque (signe de Risser) de 2. A ce stade d'évolution, quel est le traitement le plus approprié pour essayer d'arrêter la progression de la courbure ?", "full_answer": "Dans les formes juvéniles et adolescentes, le traitement dépend du degré de déformation : - 10 - 20° : observation en série. - 20-50° : orthèses thermoplastiques (Boston, Cheneau ou Milwaukee, selon l'endroit où se trouve le sommet de la courbe principale). - 40-50° : avec un Risser inférieur à 3, une arthrodèse chirurgicale serait indiquée ; avec un Risser supérieur ou égal à 3, le potentiel de croissance restant de la courbure est faible et une observation peut être envisagée au cas où elle tendrait à progresser à l'âge adulte.", "type": "CHIRURGIE ORTHOPÉDIQUE ET TRAUMATOLOGIE", "options": {"1": "Exercices de natation.", "2": "Exercices de rééducation posturale.", "3": "Corset orthopédique correcteur.", "4": "Intervention chirurgicale par arthrodèse vertébrale.", "5": null}, "correct_option": 3, "explanations": {"1": {"exist": false, "char_ranges": [], "word_ranges": [], "text": ""}, "2": {"exist": false, "char_ranges": [], "word_ranges": [], "text": ""}, "3": {"exist": true, "char_ranges": [[127, 257]], "word_ranges": [[23, 42]], "text": "20-50° : orthèses thermoplastiques (Boston, Cheneau ou Milwaukee, selon l'endroit où se trouve le sommet de la courbe principale)."}, "4": {"exist": true, "char_ranges": [[269, 344]], "word_ranges": [[45, 57]], "text": "avec un Risser inférieur à 3, une arthrodèse chirurgicale serait indiquée ;"}, "5": {"exist": false, "char_ranges": [], "word_ranges": [], "text": ""}}}
{"id": 175, "year": 2013, "question_id_specific": 53, "full_question": "Un homme de 71 ans, serveur à la retraite, non-fumeur, qui consulte pour des ronflements nocturnes. Le patient a un indice de masse corporelle de 31,5 kg/m2 et signale une légère somnolence diurne postprandiale (échelle de somnolence d'Epworth de 3) sans autres symptômes, signes ou antécédents cliniques pertinents. L'indice d'apnée-hypopnée obtenu lors de la polysomnographie était de 18. Compte tenu de ces données, quelle est la mesure la plus appropriée à adopter chez ce patient ?", "full_answer": "Avec un indice d'apnée-hypopnée inférieur à 30, en l'absence de symptômes frappants de SAHOS et en l'absence de professions à risque telles que chauffeur professionnel, etc., la bonne chose à faire est de contrôler le poids et le reste des mesures hygiéno-diététiques, avec un suivi périodique.", "type": "PNEUMOLOGIE", "options": {"1": "Mesures hygiéno-diététiques et contrôle de l'évolution.", "2": "Uvulopalatopharyngoplastie.", "3": "CPAP nasal (pression positive continue).", "4": "Ventilation mécanique non invasive.", "5": "Oxygénothérapie la nuit."}, "correct_option": 1, "explanations": {"1": {"exist": true, "char_ranges": [[0, 294]], "word_ranges": [[0, 46]], "text": "Avec un indice d'apnée-hypopnée inférieur à 30, en l'absence de symptômes frappants de SAHOS et en l'absence de professions à risque telles que chauffeur professionnel, etc., la bonne chose à faire est de contrôler le poids et le reste des mesures hygiéno-diététiques, avec un suivi périodique."}, "2": {"exist": false, "char_ranges": [], "word_ranges": [], "text": ""}, "3": {"exist": false, "char_ranges": [], "word_ranges": [], "text": ""}, "4": {"exist": false, "char_ranges": [], "word_ranges": [], "text": ""}, "5": {"exist": false, "char_ranges": [], "word_ranges": [], "text": ""}}}
{"id": 91, "year": 2012, "question_id_specific": 35, "full_question": "Une femme de 26 ans a présenté une incontinence fécale après un accouchement instrumental prolongé. Une échographie endoanale a été réalisée et une section du sphincter anal externe de 30º d'amplitude a été observée. L'étude électrophysiologique montre une innervation normale. Quel est le traitement le plus approprié ?", "full_answer": "La réponse est 3 : sphinctéroplastie chirurgicale. 1 n'est pas efficace, 2 n'est pas efficace pour une lésion organique avec électromyographie normale, 4 n'est pas nécessaire car la lésion ne concerne que le sphincter externe, 5 n'est pas indiqué pour une lésion qui peut être réparée par chirurgie avec de bons résultats.", "type": "CHIRURGIE GÉNÉRALE", "options": {"1": "Traitement médical avec règles hygiéniques et diététiques.", "2": "Rétroaction biologique du sphincter.", "3": "Sphinctéroplastie chirurgicale.", "4": "Réparation chirurgicale du plancher pelvien.", "5": "Sphincter anal artificiel."}, "correct_option": 3, "explanations": {"1": {"exist": true, "char_ranges": [[51, 71]], "word_ranges": [[7, 11]], "text": "1 n'est pas efficace,"}, "2": {"exist": true, "char_ranges": [[73, 151]], "word_ranges": [[11, 22]], "text": "2 n'est pas efficace pour une lésion organique avec électromyographie normale,"}, "3": {"exist": false, "char_ranges": [], "word_ranges": [], "text": ""}, "4": {"exist": true, "char_ranges": [[152, 226]], "word_ranges": [[22, 35]], "text": "4 n'est pas nécessaire car la lésion ne concerne que le sphincter externe,"}, "5": {"exist": true, "char_ranges": [[227, 322]], "word_ranges": [[35, 52]], "text": "5 n'est pas indiqué pour une lésion qui peut être réparée par chirurgie avec de bons résultats."}}}
{"id": 65, "year": 2011, "question_id_specific": 218, "full_question": "Un homme de 56 ans, diabétique et en surpoids modéré, présente une discrète hépatomégalie. Le patient indique qu'il n'est pas un buveur régulier et qu'il ne boit du vin qu'exceptionnellement. Une biopsie du foie montre des accumulations de vacuoles cytoplasmiques claires dans 50 % des hépatocytes. De multiples foyers inflammatoires avec des neutrophiles, une dégénérescence hyaline de Mallory et une fibrose autour des veinules hépatiques terminales. Cochez la bonne réponse dans ce cas :", "full_answer": "Les changements décrits dans l'énoncé de la question correspondent à une situation précirrhotique précoce. Bien que l'alcool soit la substance toxique la plus fréquemment à l'origine de cette affection, il n'en est pas la seule cause. Le diabète sucré provoque des altérations hépatiques similaires : accumulation de graisse dans les hépatocytes, fibrose centroacinaire, infiltration inflammatoire par des neutrophiles polymorphonucléaires et dégénérescence hyaline de la malléole périnucléaire. Compte tenu de ces éléments, nous pouvons écarter la deuxième question (il s'agit d'accumulations de graisses intracellulaires et non de glycogène) et la quatrième (si le diabète provoque également une telle situation, il n'y a pas lieu de se méfier du patient). Sur la base du primum non nocere et du fait qu'il n'est pas particulièrement important pour le traitement de savoir s'il s'agit d'une maladie focale ou généralisée, il n'est pas nécessaire de refaire une biopsie du foie, qui n'apporterait pas d'informations supplémentaires, et la première question est écartée. Heureusement pour le patient hypothétique, dans la phase initiale, ces changements sont réversibles lorsque la noxa disparaît, de sorte qu'il ne reste que la réponse n° 5.", "type": "ANATOMIE PATHOLOGIQUE", "options": {"1": "La biopsie doit être répétée pour vérifier si les changements sont diffus.", "2": "Le diagnostic est celui d'une glycogénose hépatique associée à un diabète.", "3": "Les changements histologiques sont irréversibles et conduisent à terme à une cirrhose.", "4": "La blessure est clairement causée par l'alcool, même si le patient le nie.", "5": "Il est important de réduire le poids et de contrôler correctement le diabète pour éviter le développement éventuel d'une cirrhose."}, "correct_option": 5, "explanations": {"1": {"exist": true, "char_ranges": [[759, 1070]], "word_ranges": [[108, 156]], "text": "Sur la base du primum non nocere et du fait qu'il n'est pas particulièrement important pour le traitement de savoir s'il s'agit d'une maladie focale ou généralisée, il n'est pas nécessaire de refaire une biopsie du foie, qui n'apporterait pas d'informations supplémentaires, et la première question est écartée."}, "2": {"exist": true, "char_ranges": [[235, 758]], "word_ranges": [[36, 108]], "text": "Le diabète sucré provoque des altérations hépatiques similaires : accumulation de graisse dans les hépatocytes, fibrose centroacinaire, infiltration inflammatoire par des neutrophiles polymorphonucléaires et dégénérescence hyaline de la malléole périnucléaire. Compte tenu de ces éléments, nous pouvons écarter la deuxième question (il s'agit d'accumulations de graisses intracellulaires et non de glycogène) et la quatrième (si le diabète provoque également une telle situation, il n'y a pas lieu de se méfier du patient)."}, "3": {"exist": false, "char_ranges": [], "word_ranges": [], "text": ""}, "4": {"exist": true, "char_ranges": [[235, 758]], "word_ranges": [[36, 108]], "text": "Le diabète sucré provoque des altérations hépatiques similaires : accumulation de graisse dans les hépatocytes, fibrose centroacinaire, infiltration inflammatoire par des neutrophiles polymorphonucléaires et dégénérescence hyaline de la malléole périnucléaire. Compte tenu de ces éléments, nous pouvons écarter la deuxième question (il s'agit d'accumulations de graisses intracellulaires et non de glycogène) et la quatrième (si le diabète provoque également une telle situation, il n'y a pas lieu de se méfier du patient)."}, "5": {"exist": true, "char_ranges": [[1114, 1242]], "word_ranges": [[161, 183]], "text": "dans la phase initiale, ces changements sont réversibles lorsque la noxa disparaît, de sorte qu'il ne reste que la réponse n° 5."}}}
{"id": 50, "year": 2011, "question_id_specific": 160, "full_question": "Une petite fille de 5 ans, totalement asymptomatique, qui se rend chez le pédiatre pour un contrôle régulier. Elle consulte car sa mère a été opérée d'une tumeur de la thyroïde et sa grand-mère est décédée il y a 10 ans d'un phéochromocytome. L'approche thérapeutique prioritaire que nous adopterons sera la suivante :", "full_answer": "La bonne réponse est 4. Il me semble qu'il s'agit davantage d'une question endocrinienne que pédiatrique. Les porteurs du proto-oncogène ont généralement des tumeurs thyroïdiennes précoces, de sorte que la thyroïdectomie radicale est généralement recommandée avant l'âge de 5 ans. J'ai évidemment dû faire des recherches à ce sujet... ce n'est pas très fréquent dans notre pratique habituelle.", "type": "PÉDIATRIE", "options": {"1": "Examen physique approfondi avec contrôle de la tension artérielle pour déterminer si l'enfant peut être atteint d'une maladie familiale.", "2": "Analyse biochimique avec la calcitonine. Si les résultats sont normaux, aucune autre surveillance n'est nécessaire à cet âge.", "3": "Contrôle annuel des taux de calcitonine par stimulation à la pentagastrine et, en cas d'élévation, indication d'une thyroïdectomie prophylactique.", "4": "Une étude génétique de la mutation du proto-oncogène RET c634 et, en cas de résultat positif, une thyroïdectomie radicale prophylactique seront effectuées à cet âge.", "5": "Une étude génétique de la mutation du proto-oncogène RET c634, des taux de calcitonine et une FNA (aspiration à l'aiguille fine) et, en cas de résultat positif, une thyroïdectomie radicale seront réalisées."}, "correct_option": 4, "explanations": {"1": {"exist": false, "char_ranges": [], "word_ranges": [], "text": ""}, "2": {"exist": false, "char_ranges": [], "word_ranges": [], "text": ""}, "3": {"exist": false, "char_ranges": [], "word_ranges": [], "text": ""}, "4": {"exist": true, "char_ranges": [[106, 280]], "word_ranges": [[16, 40]], "text": "Les porteurs du proto-oncogène ont généralement des tumeurs thyroïdiennes précoces, de sorte que la thyroïdectomie radicale est généralement recommandée avant l'âge de 5 ans."}, "5": {"exist": false, "char_ranges": [], "word_ranges": [], "text": ""}}}
{"id": 326, "year": 2016, "question_id_specific": 173, "full_question": "Un homme de 35 ans souffre d'une brûlure électrique à haute tension (3000 volts) due à un contact direct avec un câble dans sa main gauche. À l'admission, il présente une contracture en flexion de la main, une pâleur des doigts et une absence de pouls radial et ulnaire à la palpation.", "full_answer": "3) En cas de suspicion de syndrome des loges, la mesure d'urgence invasive à prendre serait de mesurer la pression intra-compartimentale et, en fonction du résultat, de procéder à un traitement.", "type": "LES SOINS CRITIQUES, PALLIATIFS ET D'URGENCE", "options": {"1": "Bloc axillaire avec cathéter.", "2": "Scarotomie décompressive.", "3": "Contrôle de la pression intracompartimentale.", "4": "Scarectomie.", "5": null}, "correct_option": 3, "explanations": {"1": {"exist": false, "char_ranges": [], "word_ranges": [], "text": ""}, "2": {"exist": false, "char_ranges": [], "word_ranges": [], "text": ""}, "3": {"exist": true, "char_ranges": [[3, 194]], "word_ranges": [[1, 31]], "text": "En cas de suspicion de syndrome des loges, la mesure d'urgence invasive à prendre serait de mesurer la pression intra-compartimentale et, en fonction du résultat, de procéder à un traitement."}, "4": {"exist": false, "char_ranges": [], "word_ranges": [], "text": ""}, "5": {"exist": false, "char_ranges": [], "word_ranges": [], "text": ""}}}
{"id": 354, "year": 2016, "question_id_specific": 166, "full_question": "Une femme enceinte de 10 semaines s'est présentée au service des urgences avec une tension artérielle de 160/105 mmHg. Elle était en bon état général et ne souffrait que de légers maux de tête, raison pour laquelle sa tension artérielle a été prise. Après 4 heures de repos, la tension artérielle était de 150/95 mmHg. L'hémogramme était normal et la protéinurie négative. De quel type d'hypertension souffre-t-il ?", "full_answer": "L'hypertension chronique est définie comme une hypertension présente avant la grossesse ou diagnostiquée avant la 20e semaine de gestation. Dans ce cas, la patiente est enceinte de 10 semaines, la bonne réponse est donc 3.", "type": "GYNÉCOLOGIE ET OBSTÉTRIQUE", "options": {"1": "Pré-éclampsie modérée.", "2": "Hypertension induite par la grossesse.", "3": "L'hypertension chronique.", "4": "Éclampsie.", "5": null}, "correct_option": 3, "explanations": {"1": {"exist": false, "char_ranges": [], "word_ranges": [], "text": ""}, "2": {"exist": false, "char_ranges": [], "word_ranges": [], "text": ""}, "3": {"exist": true, "char_ranges": [[0, 222]], "word_ranges": [[0, 35]], "text": "L'hypertension chronique est définie comme une hypertension présente avant la grossesse ou diagnostiquée avant la 20e semaine de gestation. Dans ce cas, la patiente est enceinte de 10 semaines, la bonne réponse est donc 3."}, "4": {"exist": false, "char_ranges": [], "word_ranges": [], "text": ""}, "5": {"exist": false, "char_ranges": [], "word_ranges": [], "text": ""}}}
{"id": 531, "year": 2021, "question_id_specific": 58, "full_question": "Chez un patient présentant une lésion ulcérovégétante sur la gencive mandibulaire au niveau de la région molaire de 5 mois d'évolution, l'apparition d'une hypoesthésie dans la région du menton doit être écartée :", "full_answer": "Les paresthésies mentonniennes (signe de Roger) sont associées à des métastases osseuses (pas paranéoplasiques mais par infiltration directe). Dans ce cas, ce qu'ils décrivent est une tumeur buccale se développant localement et infiltrant le canal mandibulaire, je pense.", "type": "ONCOLOGIE", "options": {"1": "Neuropathie paranéoplasique.", "2": "Le diagnostic de sarcoïdose avec paralysie faciale \"sur la balance\".", "3": "Infection par le virus de l'herpès chez les patients immunodéprimés.", "4": "Diagnostic de carcinome avec infiltration du canal mandibulaire.", "5": null}, "correct_option": 4, "explanations": {"1": {"exist": false, "char_ranges": [], "word_ranges": [], "text": ""}, "2": {"exist": false, "char_ranges": [], "word_ranges": [], "text": ""}, "3": {"exist": false, "char_ranges": [], "word_ranges": [], "text": ""}, "4": {"exist": true, "char_ranges": [[0, 271]], "word_ranges": [[0, 38]], "text": "Les paresthésies mentonniennes (signe de Roger) sont associées à des métastases osseuses (pas paranéoplasiques mais par infiltration directe). Dans ce cas, ce qu'ils décrivent est une tumeur buccale se développant localement et infiltrant le canal mandibulaire, je pense."}, "5": {"exist": false, "char_ranges": [], "word_ranges": [], "text": ""}}}
{"id": 93, "year": 2012, "question_id_specific": 31, "full_question": "Un garçon de 17 ans atteint d'une maladie de Crohn avec atteinte colique depuis 2 ans, sous traitement d'entretien par azathioprine, consulte pour l'apparition depuis 5 jours de nodules sous-cutanés rouge violacé, chauds, douloureux, bilatéraux dans la région prétibiale, associés à une augmentation du nombre de selles et à des douleurs abdominales. L'approche la plus appropriée dans ce cas est la suivante :", "full_answer": "Les points 2 et 4 sont exclus en soi. Ils expliquent l'érythème noueux, et nous recommandons donc un repos relatif (mais les antidépresseurs n'ont aucun sens, et nous excluons donc également le point 1)). Bien que l'érythème noueux puisse être secondaire à un processus malin, il est plus souvent dû à la maladie de Crohn elle-même (et chez un garçon de 17 ans, ce serait extrêmement rare), donc si l'affection est également accompagnée d'une poussée de maladie inflammatoire de l'intestin, la dernière option (5) semble la plus correcte.", "type": "DERMATOLOGIE, VÉNÉRÉOLOGIE ET CHIRURGIE PLASTIQUE", "options": {"1": "Recommandez un repos relatif et des compresses chaudes sur les deux jambes et ajoutez un traitement antidépresseur.", "2": "Effectuez des biopsies sur des zones de peau éloignées des zones blessées et prescrivez des analgésiques opioïdes dès le début.", "3": "Suspecter l'existence d'une lésion tumorale intestinale maligne comme facteur déclenchant du processus cutané.", "4": "Suspecter une ischémie bilatérale des membres inférieurs d'origine médicamenteuse.", "5": "Adaptation du traitement des maladies intestinales."}, "correct_option": 5, "explanations": {"1": {"exist": true, "char_ranges": [[38, 204]], "word_ranges": [[9, 34]], "text": "Ils expliquent l'érythème noueux, et nous recommandons donc un repos relatif (mais les antidépresseurs n'ont aucun sens, et nous excluons donc également le point 1))."}, "2": {"exist": false, "char_ranges": [], "word_ranges": [], "text": ""}, "3": {"exist": true, "char_ranges": [[205, 389]], "word_ranges": [[34, 67]], "text": "Bien que l'érythème noueux puisse être secondaire à un processus malin, il est plus souvent dû à la maladie de Crohn elle-même (et chez un garçon de 17 ans, ce serait extrêmement rare),"}, "4": {"exist": false, "char_ranges": [], "word_ranges": [], "text": ""}, "5": {"exist": true, "char_ranges": [[396, 538]], "word_ranges": [[68, 88]], "text": "si l'affection est également accompagnée d'une poussée de maladie inflammatoire de l'intestin, la dernière option (5) semble la plus correcte."}}}
{"id": 390, "year": 2016, "question_id_specific": 188, "full_question": "Un homme de 80 ans a été amené aux urgences par la police après avoir été retrouvé perdu dans la rue à plusieurs pâtés de maisons de son domicile, la nuit. Il dit à l'urgentiste : \"Je me sens bien, je marchais et je me suis perdu\". L'examen, par système, est anodin. Interrogé sur ses médicaments, il ne s'en souvient pas. Son dossier médical mentionne du glipizide, de l'aténolol et de l'acénocoumarol. Elle s'est rendue trois fois aux urgences au cours des trois derniers mois pour des \"vertiges\", des douleurs lombaires et un INR de 5. Elle se dit autonome dans les activités de base et instrumentales de la vie quotidienne. Il n'y a aucune preuve de l'existence d'une famille ou d'amis proches. Plusieurs références du service social de la région indiquent qu'elle refuse toute aide ou visite médicale ou infirmière. L'examen physique révèle une température de 37,7, un IMC de 16, une tension artérielle de 160/90, une fréquence cardiaque de 90 et un taux de SO2 de 99 %. Il présente une perte globale de masse musculaire. Il est habillé en robe de chambre et sent l'urine. Il refuse de répondre aux questions du Minimental. Les examens de laboratoire montrent 13 000 leucocytes, une glycémie à 300, une hémoglobine glycosylée à 11 %, un INR à 0,9, des urines à plus de 50 leucocytes/champ, des nitrites+ et une culture d'urine > 100 000 bacilles Gram négatif. Il a été admis à l'hôpital, a reçu des antibiotiques, de l'insuline et a repris l'acénocoumarol avec une bonne évolution. Son état est stable et il souhaite rentrer chez lui. L'infirmière du service dit qu'il est incapable de s'auto-administrer de l'insuline. Lequel des éléments suivants est l'étape suivante pour déterminer la sécurité du patient à domicile ?", "full_answer": "L'évaluation de la capacité ne consiste pas à priver le patient de son autonomie, mais à s'assurer qu'il comprend et est conscient des conséquences de ses décisions. Il s'agit en fait d'une mesure de protection pour les patients les plus fragiles afin de leur assurer les meilleurs soins. Elle n'est pas réalisée par un psychiatre, mais par le médecin traitant. Il existe plusieurs tests ou entretiens semi-structurés pour évaluer la capacité des patients, et tous les médecins devraient connaître ces instruments. Il est important que ce soit le médecin traitant qui procède à cette évaluation, car c'est lui qui informe préalablement le patient (consentement éclairé) du projet thérapeutique et qui est le mieux à même d'évaluer si le patient a tout compris : quel est le problème médical, quel est le traitement proposé, les éventuelles alternatives, l'acceptation ou le refus de l'une d'entre elles et pourquoi, les conséquences de ces décisions (ainsi que celles du refus du traitement), voire l'évaluation de la psychopathologie susceptible de faire l'objet d'un PIC en psychiatrie.", "type": "PSYCHIATRIE", "options": {"1": "Adressez-vous aux services de soins primaires et aux services sociaux de la région pour déterminer la sécurité à domicile.", "2": "Évaluation formelle de la capacité à prendre des décisions concernant sa santé.", "3": "Évaluation visant à exclure la démence.", "4": "Évaluation visant à écarter la possibilité d'une dépression.", "5": null}, "correct_option": 2, "explanations": {"1": {"exist": false, "char_ranges": [], "word_ranges": [], "text": ""}, "2": {"exist": true, "char_ranges": [[184, 288]], "word_ranges": [[31, 48]], "text": "d'une mesure de protection pour les patients les plus fragiles afin de leur assurer les meilleurs soins."}, "3": {"exist": false, "char_ranges": [], "word_ranges": [], "text": ""}, "4": {"exist": false, "char_ranges": [], "word_ranges": [], "text": ""}, "5": {"exist": false, "char_ranges": [], "word_ranges": [], "text": ""}}}
{"id": 577, "year": 2022, "question_id_specific": 105, "full_question": "Un homme de 45 ans est amené à la clinique par ses proches pour une année de perte de mémoire, qui a progressé au point de nécessiter une aide pour certaines activités de la vie quotidienne. Il a des antécédents de migraines et quelques épisodes de focalité neurologique autolimitée pour lesquels il n'a jamais consulté. L'examen neuropsychologique est compatible avec une démence initiale et l'IRM cérébrale montre une leucoencéphalopathie sévère. Quel examen faut-il demander ?", "full_answer": "Ils décrivent un cas caractéristique de CADASIL (Cerebral Autosomal Dominant Arteriopathy with Sub-cortical Infarcts and Leukoencephalopathy). Il s'agit d'une maladie génétique à transmission autosomique dominante due à une mutation du gène NOTCH3. Cette mutation provoque une atteinte vasculaire des petits vaisseaux qui génère une leucoencéphalopathie sévère à un stade précoce, entraînant une variété de symptômes. Les migraines, les maladies cérébrovasculaires et les troubles cognitifs précoces sont les plus fréquents, comme le montre le cas clinique de la question.", "type": "NEUROLOGIE", "options": {"1": "Ponction lombaire pour analyser les protéines amyloïdes et tau dans le LCR.", "2": "Étude génétique pour la préséniline 1 (PSEN1).", "3": "Étude génétique pour NOTCH3.", "4": "Étude génétique pour la progranuline.", "5": null}, "correct_option": 3, "explanations": {"1": {"exist": false, "char_ranges": [], "word_ranges": [], "text": ""}, "2": {"exist": false, "char_ranges": [], "word_ranges": [], "text": ""}, "3": {"exist": true, "char_ranges": [[0, 248]], "word_ranges": [[0, 32]], "text": "Ils décrivent un cas caractéristique de CADASIL (Cerebral Autosomal Dominant Arteriopathy with Sub-cortical Infarcts and Leukoencephalopathy). Il s'agit d'une maladie génétique à transmission autosomique dominante due à une mutation du gène NOTCH3."}, "4": {"exist": false, "char_ranges": [], "word_ranges": [], "text": ""}, "5": {"exist": false, "char_ranges": [], "word_ranges": [], "text": ""}}}
{"id": 615, "year": 2022, "question_id_specific": 118, "full_question": "Un homme de 41 ans consulte pour des gonalgies depuis plusieurs jours. Lors de l'examen, le test de Thessaly (douleur lors des mouvements de rotation interne et externe, genou fléchi) a été réalisé et s'est révélé positif. Parmi les lésions suivantes, laquelle est la plus probable ?", "full_answer": "Réponse correcte 1 : Le test de Thessaly est une manœuvre d'examen du ménisque. L'énoncé se concentre exclusivement sur cette partie de l'examen et explique également qu'il est effectivement positif, ce qui permet de suspecter une lésion méniscale. \"Le test de Thessaly est une reproduction dynamique de la transmission de la charge dans l'articulation du genou et est effectué à 5° et 20° de flexion. L'examinateur soutient le patient en lui tenant les mains tendues pendant qu'il se tient à plat sur le sol. Le patient effectue ensuite trois rotations internes et externes du genou et du corps, en maintenant le genou en légère flexion (5°). La même procédure est ensuite effectuée avec le genou fléchi à 20°. Les patients chez qui l'on suspecte une déchirure méniscale ressentent une gêne au niveau de l'interligne articulaire médial ou latéral et peuvent avoir une sensation de blocage ou d'accrochage. La théorie qui sous-tend ce test est que, par cette manœuvre, le genou présentant une déchirure méniscale est soumis à des conditions de charge excessives et qu'il est presque certain qu'il présentera les mêmes symptômes que ceux rapportés par le patient. Le test est toujours effectué en premier lieu sur le genou normal afin que le patient puisse être formé, notamment en ce qui concerne la manière de maintenir le genou à 5° puis à 20° de flexion et de reconnaître, par comparaison, un éventuel résultat positif sur le genou symptomatique.\" Mauvaise réponse 2 : à aucun moment ils ne décrivent de manœuvres exploratoires du ligament croisé antérieur, ni ne mentionnent de traumatisme qui aurait pu le léser. Mauvaise réponse 3 : à aucun moment ils ne décrivent de manœuvres exploratoires du ligament croisé postérieur, ni ne mentionnent de traumatisme qui aurait pu le blesser. Mauvaise réponse 4 : bien que cela puisse se produire, il ne faut pas s'attendre à ce que cela se produise chez une personne âgée de 41 ans dont l'état clinique s'est établi de manière aiguë.", "type": "TRAUMATOLOGIE", "options": {"1": "Lésion méniscale.", "2": "Rupture du ligament croisé antérieur.", "3": "Rupture du ligament croisé postérieur.", "4": "Lésions dues à l'arthropathie dégénérative.", "5": null}, "correct_option": 1, "explanations": {"1": {"exist": true, "char_ranges": [[0, 248]], "word_ranges": [[0, 38]], "text": "Réponse correcte 1 : Le test de Thessaly est une manœuvre d'examen du ménisque. L'énoncé se concentre exclusivement sur cette partie de l'examen et explique également qu'il est effectivement positif, ce qui permet de suspecter une lésion méniscale."}, "2": {"exist": true, "char_ranges": [[1451, 1617]], "word_ranges": [[238, 265]], "text": "Mauvaise réponse 2 : à aucun moment ils ne décrivent de manœuvres exploratoires du ligament croisé antérieur, ni ne mentionnent de traumatisme qui aurait pu le léser."}, "3": {"exist": true, "char_ranges": [[1618, 1787]], "word_ranges": [[265, 292]], "text": "Mauvaise réponse 3 : à aucun moment ils ne décrivent de manœuvres exploratoires du ligament croisé postérieur, ni ne mentionnent de traumatisme qui aurait pu le blesser."}, "4": {"exist": true, "char_ranges": [[1788, 1979]], "word_ranges": [[292, 328]], "text": "Mauvaise réponse 4 : bien que cela puisse se produire, il ne faut pas s'attendre à ce que cela se produise chez une personne âgée de 41 ans dont l'état clinique s'est établi de manière aiguë."}, "5": {"exist": false, "char_ranges": [], "word_ranges": [], "text": ""}}}
{"id": 456, "year": 2018, "question_id_specific": 200, "full_question": "Un homme de 67 ans, diabétique et asthmatique, atteint d'un glaucome primaire à angle ouvert aux deux yeux, présente une pression oculaire de 34 mmHg à l'œil droit et de 31 mmHg à l'œil gauche. Lequel des groupes pharmacologiques suivants est le plus indiqué comme premier choix pour le traitement hypotenseur ?", "full_answer": "On nous demande quel est le traitement de premier choix pour le glaucome. Dans le passé, les bêta-bloquants étaient le premier choix, mais aujourd'hui ils ont été remplacés par le groupe qui abaisse le plus la pression intraoculaire : les prostaglandines. Leur efficacité et leur posologie plus pratique (une fois par jour) en font généralement le groupe de choix. Les bêta-bloquants seraient le deuxième choix dans les cas où les effets secondaires des prostaglandines (changement de couleur des yeux, augmentation de la taille des cils) nous amènent à les laisser de côté dans un premier temps. Le patient étant asthmatique, les bêta-bloquants sont à exclure (mauvaise réponse 2). Par ailleurs, comme nous avons besoin d'une baisse significative de la pression intraoculaire car les chiffres sont élevés (34 et 31), nous pouvons également écarter les inhibiteurs de l'anhydrase carbonique et les alpha-2 agonistes (options 1 et 3 erronées). Enfin, nous pouvons choisir le groupe qui peut abaisser le plus la pression intraoculaire. La réponse 4 est correcte. Et puis quelques nuances dans l'énoncé. Il serait plus correct de parler de pression intraoculaire et non de \"pression oculaire\". Et la formulation pourrait être améliorée afin de ne pas répéter le mot \"tel que\" dans la dernière phrase.", "type": "OPHTHALMOLOGIE", "options": {"1": "Inhibiteurs de l'anhydrase carbonique.", "2": "Bêta-bloquants.", "3": "Agonistes alpha-2.", "4": "Dérivés de la prostaglandine.", "5": null}, "correct_option": 4, "explanations": {"1": {"exist": true, "char_ranges": [[683, 942]], "word_ranges": [[108, 147]], "text": "Par ailleurs, comme nous avons besoin d'une baisse significative de la pression intraoculaire car les chiffres sont élevés (34 et 31), nous pouvons également écarter les inhibiteurs de l'anhydrase carbonique et les alpha-2 agonistes (options 1 et 3 erronées)."}, "2": {"exist": true, "char_ranges": [[597, 682]], "word_ranges": [[96, 108]], "text": "Le patient étant asthmatique, les bêta-bloquants sont à exclure (mauvaise réponse 2)."}, "3": {"exist": true, "char_ranges": [[683, 942]], "word_ranges": [[108, 147]], "text": "Par ailleurs, comme nous avons besoin d'une baisse significative de la pression intraoculaire car les chiffres sont élevés (34 et 31), nous pouvons également écarter les inhibiteurs de l'anhydrase carbonique et les alpha-2 agonistes (options 1 et 3 erronées)."}, "4": {"exist": true, "char_ranges": [[74, 255]], "word_ranges": [[13, 41]], "text": "Dans le passé, les bêta-bloquants étaient le premier choix, mais aujourd'hui ils ont été remplacés par le groupe qui abaisse le plus la pression intraoculaire : les prostaglandines."}, "5": {"exist": false, "char_ranges": [], "word_ranges": [], "text": ""}}}
{"id": 484, "year": 2020, "question_id_specific": 175, "full_question": "Un jeune homme de 18 ans arrive au service des urgences de l'hôpital, transporté par une unité de réanimation. Il a été renversé par une voiture alors qu'il faisait du vélo. À son arrivée, il présente une fréquence cardiaque de 115 bpm, une pression artérielle de 110/75 mmHg, un pouls radial palpable, un remplissage capillaire de 2,5 secondes et une fréquence respiratoire de 25 tr/min. Que nous apprennent ces données ?", "full_answer": "Techniquement, le patient présente une hémorragie de grade II selon la classification ATLS, ce qui signifie en principe une perte de sang comprise entre 750 et 1500 ml et l'action indiquée est la réanimation avec des cristalloïdes ; toutefois, dans les lignes directrices pour les soins aux patients polytraumatisés graves, l'utilisation précoce de dérivés plasmatiques est de plus en plus recommandée, en particulier dans les cas limites comme celui-ci et chez les jeunes patients, qui peuvent ne pas être en mesure de voir la gravité de la perte de sang jusqu'à ce qu'elle devienne beaucoup plus importante. J'hésiterais donc entre les options 1 et 2.", "type": "SOINS INTENSIFS", "options": {"1": "Il a perdu 15 à 30 % de son volume sanguin et a besoin d'un remplacement de volume.", "2": "Il a perdu plus de 30 % de son volume sanguin et doit être remplacé par du sang et du volume.", "3": "Il a perdu plus de 40 % de son volume sanguin et devra subir un traitement chirurgical.", "4": "Il a perdu plus de 50 % de sa volémie et a besoin d'une substitution de volume et de sang.", "5": null}, "correct_option": 1, "explanations": {"1": {"exist": false, "char_ranges": [], "word_ranges": [], "text": ""}, "2": {"exist": false, "char_ranges": [], "word_ranges": [], "text": ""}, "3": {"exist": false, "char_ranges": [], "word_ranges": [], "text": ""}, "4": {"exist": false, "char_ranges": [], "word_ranges": [], "text": ""}, "5": {"exist": false, "char_ranges": [], "word_ranges": [], "text": ""}}}